Anda di halaman 1dari 72

Critique Goodies Evazon 2013

Framework/Etc

Ontology 1st
Ontology determines the results of political science inquiries
Stanley 12 Liam Stanley, Rethinking the Definition and Role of Ontology in
Political Science, Politics, Vol. 32 (2), p. 93-99, 2012
In contrast, ontology in political science concerns the implicit and simplifying
assumptions about political reality that underpin explanations of political
phenomena. The analysis and discussion of ontological assumptions should therefore be considered a
second-order endeavour. Consequently, ontology should not be defined as the world as it actually is but instead as

In order to explain the political world, it is


necessary whether implicitly or explicitly to commit to a certain
(ontological) view of what is possible in social reality. Thus, explanations,
theories and approaches all contain ontological assumptions. Part of political
the world as political scientists assume it to be.

science is the parsimonycomplexity trade-off (see Hay, 2002, pp. 2937). This trade-off is an epistemological
concern since justifications for parsimony or complexity are often driven by the kind of knowledge desired. While
rational choice researchers may be keen to make simplified ontological assumptions about utility-maximising actors
in order to build more parsimonious explanations, some cultural anthropologists disregard the idea of parsimonious

even a cultural
anthropologist will have to make some form of simplifying (ontological)
assumption, perhaps that culture and identity constitute social reality. No matter where placed
on the parsimoniouscomplexity trade-off, ontological assumptions are
logically simplifying to some extent. Lay explanations highlight the role of
simplifying assumptions. For instance, a popular factor often given to explain
the electoral success of Barack Obama is that of his excellent oratory
skills and charisma. For this explanation to be possible it is assumed that
actors personal skills and abilities can, in part, cause important political
events. In other words, it is committing to a position within the structure and
agency debate. Ontological assumptions cannot be divorced from
epistemological and methodological concerns. No ontologically neutral
epistemological claims can be made: to commit oneself to an epistemology is also to commit
explanations altogether in favour of contextually specific rich interpretations. However,

oneself to a position on a range of ontological issues (Hay, 2007, p. 117). The directional dependence model (Hay,
2002) highlights this relationship. Showing this connection is important for the argument made later on; that is,

to

commit to an epistemological position is to commit (implicitly) to an


ontological position. As the ideal-type, the directional dependence model prescribes that ontological

assumptions should logically precede epistemological and methodological assumptions creating a path of
dependency, meaning that all three sets of assumptions are conceptually linked and realistic. Likewise, within this
model it is deemed logically impossible to make an ontologically neutral epistemological decision due to this
directional dependence. However, what this means in practice is rather simpler and different. For example, if a
researcher were inclined to generate causal knowledge claims, possibly through the use of explanatory quantitative
methods, then it would not be logical for this researcher then to assert that political reality is constituted through
meaning and language. This model relates to the parsimonycomplexity trade-off. The chances of creating a
simplified generalisable explanation of politics (an epistemological assumption) is increased if simplifying

all political science


explanations are underpinned by necessarily simplifying assumptions
about the nature of reality, which in turn may emerge from epistemological choices. However, the
assumptions about social reality (an ontological decision) are made. Therefore,

literature problematically defines ontology as what exists in reality. This implies that ontological dualisms such as
structureagency may allow researchers some first-order analytical purchase. In contrast, a distinctively secondorder definition of ontology would not make similar implications. Instead, ontological dualisms would be considered
as useful heuristic devices for making sense of the assumptions behind existing explanations and approaches. This
is consistent with what I have described as second-order political science.

Epistemology 1st
Epistemic commitments determine ontological outlooks and
predictive resultsprior critique is necessary to stave off banal
instrumentality and institutional bias
Stanley 12 Liam Stanley, Rethinking the Definition and Role of Ontology in
Political Science, Politics, Vol. 32 (2), p. 93-99, 2012

The literature says very little directly on the subject of how ontological assumptions emerge (Hay, 2005, p. 41). In
many ways, this is unsurprising, particularly because it is not possible to conduct such an analysis without first
committing to a series of ontological and epistemological assumptions (which I, of course, must also make in the
remainder of the article). This inescapable irony is noted, but should not prevent the endeavour altogether because
otherwise such discussions would not even be possible. Nevertheless, in regard to how political scientists make
ontological assumptions some preliminary answers can be gathered from reading the literature from in between the
lines. While the literature often rightly claims that ontological dualisms are perennial problematiques with no
solution (Hay, 2006, p. 82; Jenkins, 2005, p. 6), they also tend somewhat paradoxically to offer conceptual
refinements to these unsolvable dichotomies: e.g. structureagency (McAnulla, 2002), material ideational (e.g.

if ontological dualisms
cannot be solved then why do political scientists seek to offer increasingly
complex conceptualisations? It is presumably because such
problematiques allow political scientists to reflect on their own
assumptions, as well as the assumptions of others, and avoid making the
simplistic structuralist or intentionalist mistakes of yesteryear. Such
simplistic underpinnings should indeed be critiqued on the basis of
unrealistically limiting the potential for human agency or failing to
consider how structures favour certain actors and strategies . But the value of
Marsh, 2009), mindbody (e.g. Jenkins, 2005). So, the question becomes,

incrementally more complex conceptualisations is rarely justified through this. Furthermore, in tandem with the
ideal-type of the directional dependence, this aspect of the literature also implies that ontological assumptions
should emerge from engagement with philosophically oriented literature. Yet there is little reflection on whether this
ideal-type accurately reflects academic practice and, more importantly, whether this would have any implications
for foundations of their arguments. A second reading of the directional dependence model could also imply that
ontological assumptions sometimes derive from epistemological decisions (Hay, 2006, p. 92). This is why the

seemingly
innocuous epistemological or methodological decisions can influence
assumptions about social reality. If some ontological assumptions are inextricably tied up with
directional dependence model outlined earlier is important, because it demonstrates how

epistemological decisions, then the next step should involve the analysis of the process that, in part, gives rise to

Epistemological decisions can


influence or generate ontological assumptions. Colin Hay (Hay, 2006, p. 92) has shown
epistemological assumptions. This section aims to do just that.

the adverse affects of searching for generalisable knowledge (an epistemological decision) on the ontological

the extent of epistemological


decision-making extends beyond the understandable ambition to theorise
politics. It is sometimes influenced by academic norms emerging from the
discipline as well as regulatory and funding bodies. One possible criticism of the
literature on ontology is that it is irrelevant to the majority of political scientists who are driven by
a mundane instrumentality in which what counts is what works (Bale, 2006,
p. 102). Yet, this mundane instrumentality often manifests itself in the form of
epistemological decisions, which may then influence ontological
assumptions. Other researchers have made similar arguments, albeit not directly. For instance, Alan Bryman
assumptions and resulting explanations of rational choice theory. Yet,

(2007) argues that the quantitativequalitative paradigm wars have been superseded by a certain ontological and
epistemological pragmatism in which philosophical reflection is rendered obsolete in the pursuit of further funding
and publications. When interviewing a number of leading social scientists Bryman (2007, p. 17) found that: when
asked about how far epistemological and ontological issues concerned them, most interviewees depicted
themselves as pragmatists who felt it necessary to put aside such issues to secure funding for their research

Donovans (2005) analysis starts from


the observation that nonpolitical scientists, who often impose
interests and to publish their findings. Meanwhile, Clare

inappropriate regulation in adjudicating the strength of research, regulate


the discipline. The consequence, for Donovan, is the rise of a slave social
science in which positivism, the epistemological approach most associated with natural science,
becomes dominant despite, perhaps, the misgivings some may have over its
ontological ramifications. This suggests that ontological assumptions do not necessarily
emerge from conscious deliberation with solutions to philosophical dualisms but are inextricably linked
to a number of factors including the (perceived) epistemological biases of
regulatory and funding bodies.

Language 1st
Linguistic decisions matterthey shape symbolic interaction
with the world
Boshoff 12 Anl Boshoff, Law and Its Rhetoric of Violence, International

Journal for the Semiotics of Law, June 2012, 10.1007/s11196-012-9277-6


Two assumptions underlie my argument. First, choice of words or legal terminology is never
neutral. As Agamben suggests, the choice of term implies a specific position taken
both on the nature of a phenomenon and on the logic most suitable for
understanding it [3]. Choosing the term is crucial and, in his words, it is the proper poetic moment of
thought.2 Second, legal terminology is a powerful ideological tool; it is not a way
of describing the world, but rather, it is a way of making it. The capacity of
language, and in this case specifically of legal language, to shape the vast symbolic
landscape we live in, to draw the boundaries of what we regard as
possible and impossible, to define the grid of our knowledge is , I will argue,
often overlooked.3 From a semiological point of view the law is a system of
signs that turns reality into speech, but, and this is what I want to argue, also turns
speech into reality.

Language is prior to plan focusstructures reality and shapes


political and ethical responses
Boshoff 12 Anl Boshoff, Law and Its Rhetoric of Violence, International
Journal for the Semiotics of Law, June 2012, 10.1007/s11196-012-9277-6
The first question is whether rhetoric, or terminology, as the law calls it, really matters? Is it important, either on the
level of institutional legal organisation or on the level of everyday
singularity, to distinguish between the operative verbs: fighting poverty,
addressing the problem of poverty, or, alleviating poverty? Is it necessary and possible to
resist laws use of metaphor? We have it from Aristotle that [m]etaphor consists in giving the thing a name that belongs to something else [10]. In
semiology it is generally accepted that metaphoric thinking is not only possible and widespread, but that it may in fact be impossible for us to think

Reality is inevitably framed within systems of analogy and this


enables one to see one concept in terms of another. The ubiquity of tropes
reflects our fundamentally relational understanding of things. If
understanding is seen as a process of rendering what is unfamiliar
familiar, the use of figures of speech is seen as essential to the process . On
without metaphors.

the question whether we can escape metaphor, Roland Barthes gave a definite no, no sooner is a form seen than it must resemble something: humanity
seems doomed to analogy [as cited in [11]]. Lakoff and Johnson explain a further feature of rhetoric as persuasive speech, which brings it closer to what
we might call ideology, namely that it is for the most part invisible [12]. This implies that we are also mostly unaware of the symbolic influence and
effect, or even the existence, of these persuasive and value-laden figures of speech. Chandler [13] explains: Such transparency tends to anaesthetize us
to the way in which the culturally available stock of tropes acts as an anchor linking us to the dominant ways of thinking within our society. Our repeated
exposure to, and use of, such figures of speech subtly sustains our tacit agreement with the shared assumptions of our society.23 Barthes goes much
further than the innocence of Chandlers subtly sustaining tacit agreements, and describes a semiological process whereby a sign is appropriated,
emptied of historical meaning, and then purposely used to imply an emotively strong but conceptually empty signification. This he calls mythology, or

Myth] abolishes the complexity of human acts, it gives them the simplicity of
organises a world which
is without contradictions because it is without depth, a world wide open
and wallowing in the evident, it establishes a blissful clarity: things
appear to mean something by themselves.24 Barthes points out that the process of
second-level signification (or producing so-called myths) is never without a motive, although the
motive is often very fragmentary. It plays on the analogy between the meaning and the
concept, but this analogy is always partial, emotive and rather vague. Barthes
goes so far as to say that a complete image/analogy would exclude myth: [I]n general, myth prefers to work with poor,
incomplete images, where meaning is already relieved of its fat, and ready
for a signification, such as caricatures, pastiches, symbols, etc .25 This point can be
second-level signification. He describes the process as follows: [

essences, it does away with all dialectics, with any going back beyond what is immediately visible, it

explained by taking a sideways step in the argument and looking, albeit briefly, at Susan Sontags illustration of the potent and harmful effects of the
metaphors of violence and destruction, both on the body and on the body politic. In Illness as Metaphor and AIDS and its Metaphors [14], she aims at, in
her words, find[ing] a more truthful way of regarding illness one most purified of, more resistant to metaphoric thinking.26 Her main concerns are,
first, that the controlling metaphors of illness (cancer, in particular, as that from which she suffered), are drawn from the language of warfare and, second,
that illnesses, in turn, are used as metaphors in modern politico-legal discourse and that they thereby carry their melodramatic and simplistic rhetoric over
into the public sphere. Sontag illustrates, from first-hand experience, the militaristic and distinctly paranoid flavour of current medical terminology. A few
examples will suffice: Cancer cells do not simply multiply, they are invasive27 Cancer cells colonize from the original tumor to far sites in the body,
first setting up tiny outposts whose presence is assumed, though they cannot be detected. Rarely are the bodys defenses vigorous enough to obliterate
a tumor However radical the surgical intervention, however many scans are taken of the body landscape, most remissions are temporary; the
prospects are that tumor invasion will continue, or that rogue cells will eventually regroup and mount a new assault on the organism.28 Cancer
treatment, likewise, suggests strategies of modern military combat. During radiotherapy patients are bombarded with toxic rays and chemotherapy is
quite literally chemical warfare.29 Cancer treatment inevitably also harms or even destroys healthy cells, thereby causing collateral damage, but
strategically it is thought that nearly any damage to the body is justified for the greater good of saving the patients life. Like in war, the strategy often
does not work and the patient has to be killed to be saved from the disease.30 Looking at it from the opposite direction, medical analogies have long been
used to explain societal problems (symptoms) and to justify politico-legal intervention (treatment). Unlike earlier rhetorical use, she argues, where it was
assumed that illness/disorder can in principle be managed or treated, modern diseases like AIDS and cancer are more loathsome and fatal and thus are to
be attacked. Similarly, we no longer have localised unrest, political differences or regional instabilitywe have an all-out global attack on civilization31;
an incurable and fatal illness, which must be destroyed before it destroys the world. No political view has a monopoly on the use of this metaphor. In
speeches on the Jewish problem throughout the 1930 s, European Jewry was repeatedly analogized to syphilis and to a cancer that must be excised
chillingly, the prescribed cure was a complete surgical removal, often also cutting out the healthy tissue around it.32 Winston Churchill remarked in 1929
that the Germans transported Lenin in a sealed truck like a plague bacillus from Switzerland into Russia.33 Trotsky, often seen as the most talented
communist polemicist, described Soviet bureaucracy under Stalin as a cancerous growth on the body of the working class.34 In 1970 s USA John Dean
explained Watergate to Nixon: We have a cancer withinclose to the Presidencythats growing.35 Sontag argues, correctly in my view, that war-making
and its associated vocabulary is often used in an inappropriate and harmful way, and often for purposes of mass ideological mobilization.36 It is a useful
metaphor, or myth in Barthess vocabulary, for all sorts of ameliorative campaigns whose goals are cast as the defeat of an enemy.37 She explains
that in a capitalist society decisions and actions are normally subject to calculations based on self-interest and profitability. All-out war, however, is to
some extent exempted from the language of realistic and practical decision making. It is regarded as an emergency situation where normal rules and
safeguards are suspended, an Ausnahmenzustand, in Schmitts words. It is a situation where no sacrifice is too much, not even sacrifice of the law itself.
It is a rousing call-to-arms to fight an insidious enemy. The problem, however, is that in creating the analogy, in the process of mythification, the sign
(war/fighting) is appropriated and separated from its historical context. But it nevertheless retains some confused and shapeless associations that sustain
the fiction of a context-rich factual situation. It therefore seems possible, however absurd (if seen from outside the system), for the enemy to be an
abstract notion, such as world hunger or international terror, a notion so vague that attacking it makes no sense. It also makes it appear possible for
dealing with the huge complexities38 inherent in these problematic situations to be simplified to one-dimensional acts of attack and to reach a definite
and irreversible outcome, namely destruction.39 The question is whether it is possible, particularly for legal and political language, which is so prone to
corrupt and ostentatious justifications, to be purified of metaphor, or to, at least, show some resistance to metaphoric thinking. The question itself is
highly problematic because it assumes that there is some form of pure, untainted or innocent meaning that can be excavated from under the ideological
distortions, something akin to Habermass ideal speech situation [15]. Unfortunately however, we are confronted with Foucaults linguistic determinism
where the dominant tropes within the discourse of a particular historical period determine what can be knownconstituting the basic episteme of the age

Linguistic or semiotic frameworks create what Foucault calls the conditions of


possibility. It means that discourses, such as the legal discourse, have real effects in that they
structure the possibility of what gets included and excluded in the debate
and hence also of what gets done or remains undone. Seen like this, and as pointed out earlier,
language does not reflect reality, it does not even distort reality; it creates reality. What is more, the integrity of
this process, which is always essentially an historical meaning-making process, remains in doubt.
[16].

Facts Bad
Cold facts are worthless. We have to surpass matters of fact to
make facts matter.
Schlag 13 Pierre Schlag, Facts (The), his blog, 1/28/2013,

http://brazenandtenured.com/2013/01/28/facts-the/
But let me explain about the facts. First, notice, that the most factish of facts (apologies to Latour)
are actually factoidstrivial data bits shorn of any actual narrative. CNN had
it down cold: America has had five presidents who ate fish for breakfast .
What, I ask you, could you possibly do with that qua fact? Still, Americans like facts. It was
Joe Friday on Dragnet who first said, all we want are the facts, maam. Really? Thats all? I dont think so. He was
on a mission. He wanted facts on a mission. And we, the viewers, did too. So I have to say, as a preliminary matter,

in their most prototypical


factishisness, facts are nearly useless while in their most desirable state
they are on a missionwell, thats not an auspicious start. Things get worse. In law and social
science (thats my domain limit hereI feel really cramped) facts generally function as
poseurs. The facts, are nearly always posing as the truth about what-isactually-going-on. Facts are frequently presented as the-real-story or the
bottom line. One is no doubt supposed to conclude from this that facts
are factsthat they are the veritable bedrock of truth. But notice that
this doesnt make any sense. Notice that the bottom line is an accounting
metaphor. Consider that, the real story is an oxymoron deliberately
composed of both truth and fiction. Note that what-is-actually-going-on is a
problematic state hanging precariously on the ungrounded and
notoriously unreliable reality/appearance pair. All of this is to say, that the appeal
of getting down to the facts, (or some such thing) often rests on situating the
facts in some initially alluring rhetorical space (e.g. the real story the bottom line)
that turns out, upon further inspection, to be constructed of images,
metaphors or fictions of questionable philosophical countenance . (See,
Nietzsche, On Lies and Truth in a Non-Moral Sense) Now, its not that these metaphors, images
or fictions turn facts into non-facts. But still, I ask you: what could be more
humbling to a fact then to learn that its appeal rests upon a fiction? Not
only do facts frequently function as poseurs, but, when they are at their
most factish, theyre often not all that interesting. Factish facts dont
really tell you much of anything you want to know. Imagine a party. Here are some
exemplary factish facts: There were 19 people at the party. 9 were women. 10 were
men. While the party was happening, gravity exercised a constant force of
32 feet per second/per second. Everyone standing stayed connected to the
ground. Not the greatest narrative is it? And notice here that if you stick strictly
to the facts (if you admit only of truly factish facts) adding more of these little items will
not markedly improve your story line. (For you editors of university press books and law review
articles, please pay special attention here.) The only time facts are really interesting
(remember law and social science is the domain limit) is when theyre something more than
just the facts. Go back to the party. Heres another fact: Jill left the party with Tom. This
fact is more interesting. Well, mildly so. With this sort of fact, you can start
imagining possible implications (amorous, murderous, whathaveyou). But note that now
were no longer talking about just the facts. Were talking about facts
with implications, facts with attitude. Why then are facts ever interesting?
things already dont look too good for the facts. Indeed, the possibility that

Well, ironically its because theyre not functioning as just facts, but
something more.

IR Bad
IR scholarship is distorted by systemic certainty bias
contestation key to pragmatism and accuracy
Widmaier and Park 12 Wesley W. Widmaier, Susan Park, Differences

Beyond Theory: Structural, Strategic, and Sentimental Approaches to Normative


Change, International Studies Perspectives, Volume 13, Issue 2, pages 123134,
May 2012, DOI: 10.1111/j.1528-3585.2012.00459.x
In the security realm, Mitzen and Schweller (2011:2) have likewise argued against what they term the
uncertainty

bias in IR scholarship. While this bias treats opacity and a lack of


certainty as a source of instability, they counter that the problem does not seem
to be uncertainty but certainty. Highlighting the importance of misplaced certainty as a distinct
and common pathway to war, Mitzen and Schweller argue that misplaced certainty occurs
where decision makers are confident that they know each others
capabilities, intentions, or both; but their confidence is unwarranted yet
persists even in the face of disconfirming evidence. Such tendencies
could be seen, not least they note, in the convictions of senior US policymakers in
the run-up to the 2003 invasion of Iraq. Mitzen and Schweller elaborate by suggesting that
this stress on misplaced certainty may itself promise a restored pragmatic
relevance to IR theory, providing social theoretical underpinnings to the
Classical Realist admonition of prudence. In terms of the interplay of concerns for systemic
trends and organizational pathologies, this stress on misplaced certainty can itself
highlight the overlooked interplay intellectual and everyday beliefs . For

example, with respect to basic theories of organizational design, March and Olsen (1998:964965) stressed the
dangers of competency traps that lead agents to resist the modification of norms and impede the development of

policymakers can become


locked into a particular rule-based structure by virtue of developing
familiarity with the rules and capabilities for using them. Such
tendencies build pressures for instability as the accelerating
development of competence with particular institutional arrangements
and practices runs counter to the long-run obsolescence of such
practices in ways that engender performance crises. Moreover, to the
extent that technocratic overconfidence in utilitarian efficiency obscures
the need for popular consent, such performance crises can in turn inspire
legitimacy crises. As Rorty (1987:245246) argues, when policy debate becomes overly
reliant on the sterile jargon of quantified social sciences (maximizes
satisfaction, increases conflict, etc.), [ policymakers] either tune out, or, more
dangerously, begin to use the jargon in moral deliberation. To the extent that
new strategies, procedures, or technologies. Over time, they warned,

popular consent derived from utilitarian performance and analytic argumentation can only last as long as policy

intellectual stability ironically might be said to cause


policy instability and crisis.
keeps delivering the goods,

AT: Specific Alt Key


The demand for specificity is ludicroussmart critique is
productive by itself
Bogost 13 Ian Bogost, WELL, WHAT'S YOUR SOLUTION THEN? David Graeber on
thinking about ideas, 4/14/2013,
http://www.bogost.com/blog/well_whats_your_solution_then.shtml
Lately, it's common to see critiqueeven smart, detailed critiqueanswered with a
crass dismissal: "Well, what's your solution then?" As if the very idea of
raising a concern is invalid on its own. Among boosters, no critique is deemed
valid without a complete alternative program. This David Graeber article is about much

more than just critique, but I enjoyed it for the following response to demands for programmatic solution: Normally,

when you challenge the conventional wisdom--that the current economic and political system
is the only possible one--the first reaction you are likely to get is a demand for a detailed
architectural blueprint of how an alternative system would work, down to
the nature of its financial instruments, energy supplies, and policies of
sewer maintenance. Next, you are likely to be asked for a detailed program
of how this system will be brought into existence . Historically, this is
ridiculous. When has social change ever happened according to
someone's blueprint? It's not as if a small circle of visionaries in
Renaissance Florence conceived of something they called "capitalism,"
figured out the details of how the stock exchange and factories would
someday work, and then put in place a program to bring their visions into
reality. In fact, the idea is so absurd we might well ask ourselves how it ever
occurred to us to imagine this is how change happens to begin .

Specific proposals are ineffectivecriticism creates shifts in


the climate of ideas which spills over to policy
implementation
John 13 Peter John, Professor of Political Science and Public Policy at University
College London, Political Science, Impact and Evidence, Political Studies Review,
Volume 11, Issue 2, pages 168173, May 2013, DOI: 10.1111/1478-9302.12009
Political scientists should not expect a direct causal impact on the actions
of politicians and policy makers. They are more likely to influence the
climate of ideas, which in turn can shape public policy . In general, politicians and
policy makers are hungry for ideas and want to see themselves at the
forefront of new debates. In particular, they are interested in findings that they
and their advisers cannot create for themselves. The techniques of advanced political
science and debates within it are far from being arcane for they offer a new approach and provide robust evidence

Political scientists should not aim to be like journalists,


commentators or advisers as they will usually perform less well than these
talented practitioners. By communicating in conferences, tweeting, blogging and
public speaking they will find that politicians and civil servants will come
looking for them as well as the other way round. The article reviews studies of the
about politics and policy.

diffusion of ideas and the ways in which ideas influence politicians and other policy makers, which back up an
indirect approach to impact. There are broadly two ways whereby impact can occur. I am going to call the first
direct and the second indirect. The direct approach works like this. An academic I shall call A has a new idea or
finding that is relevant. Academic A then mounts a frenzied campaign to convince others of the importance of this
work, which involves making contact with journalists, writing to policy makers, emailing staff in think tanks and

calling up people in the know. The academic summarises the conference or journal paper in easy-to-understand
language and boils the research project down to a list of findings. Academic A accepts an invitation to talk on a local
radio show, then takes a trip to London to attend an event and hands business cards round to those who stay for
the drinks reception afterwards. Academic A decides to get to know key people in government and in the media.
Over time this rising public figure starts to become one of the great and good, getting invited to talk at round
tables and seminars. Academic A uses these venues to plug the research. This expert becomes a specialist adviser
to a Select Committee of the House of Commons, and takes the opportunity to mention the research to the
committee clerks and MPs. More media appearances and interviews follow. The indirect approach can happen in the
following way. Academic B has a new idea or finding, but just gives the paper at conferences or gatherings, talks
about it to friends and students, and gets on with the business of publishing it in a peer-reviewed journal. A PhD
student tweets the main findings, which get re-tweeted. The idea circulates round expert circles and starts to
diffuse, becoming common knowledge in overlapping academic circles and among the people who are linked into
them. A researcher for a BBC Radio 4 magazine programme, who takes pride in following academic debates, notices
the tweet, and clicks on the shortened URL to find the pdf of the paper. The researcher reads the title and abstract,
hacks through the literature review, takes a deep breath when presented with the formal model, skips a lot of the
statistical analysis, then finds the discussion of the findings to be very interesting. After musing on the conclusion
and rereading the abstract, the researcher thinks I understand this now. Nothing happens at this point, but the
researcher is talking to a friend who works on another channel and they start a conversation about what is hot in
various fields of activity. The ping pong continues for a bit, and then the first researcher says: You know what
there is this academic's research. The other researcher says: you are right, I have seen something on that too. You
could do a programme on it and take pot shots at government policy. Next day there is a planning meeting for next
year's schedule. The idea of the researcher gets piled in. Academic B is not aware of anything at this stage. The
researcher drops the academic an email and asks for a phone call. This duly happens. The academic finds the
conversation is rather like explaining something to a bright third-year undergraduate and the researcher's questions
are easy to answer. Academic B puts the phone down and goes back to the emails. Then the programme is
commissioned and then Academic B is duly invited to contribute. These hypothetical illustrations could take a
different course. Academic A might in the end get invited to be an expert who helps commission the policy.
Academic B might be a bit pompous and never get invited on to the programme. But if the basic plausibility of the

it is not straightforward to
promote one's own work when there are so many competitors out there,
such as other academics, experts and those working for pressure groups.
Just because an idea is good or interesting, it does not follow that anyone
is going to listen, no matter how well and vigorously it is argued for . The
second and more subtle lesson is that the act of direct promotion of the research
findings might put the recipients off the message. The most powerful
two scenarios is accepted then a few points follow. The first is that

impacts occur when people do not feel they are being persuaded, so the irony
might be that the more academics try to plug their work, the less likely it is that those in public life are going to be

in public life there are people like journalists


and researchers who get a buzz out of finding a new idea and presenting it
to others in a way that is simplified and has their stamp on it. Their pay-off
comes from turning a complex work of social science into something
digestible, then selling it to other audiences. Tim Harford (2006), author of The
influenced by it. The third insight is that

Underground Economist, is a good example of a journalist who adopts this approach. The fourth point is that

ideas circulate in networks, which means that they can come from many
sources, and so appear to be influential because they are being
articulated by different kinds of people across linked networks. When the
idea does not operate on its own, it can be accessed as the property of a
group of scholars or experts and will be accepted more willingly as a
consensus or emerging set of findings.

AT: Sil/Epistemic Pragmatism


Debating theoretical grounds produces practical relevance and
is the condition of possibility for their analytical pragmatism
Reus-Smit 12 Christian Reus-Smit, International Relations, Irrelevant? Dont

Blame Theory, Millennium: Journal of International Relations 40, 2012, pp. 526-541,
Sage
Firstly, whether IR is practically relevant depends , in large measure, on the kinds of
questions that animate our research. I am not referring here to the commonly held notion that we
should be addressing questions that practitioners want answered. Indeed, our work will at times be most relevant when we pursue
questions that policymakers and others would prefer left buried. My point is a different one, which I return to in greater detail below.
It is sufficient to note here that being practically relevant involves asking questions of practice; not just retrospective questions
about past practices their nature, sources and consequences but prospective questions about what human agents should do. As I
have argued elsewhere, being practically relevant means asking questions of how we, ourselves, or some other actors (states,

our ability, nay willingness, to ask such


questions is determined by the metatheoretical assumptions that
structure our research and arguments. This is partly an issue of ontology what we see
affects how we understand the conditions of action, rendering some
practices possible or impossible, mandatory or beyond the pale . If, for example, we
policymakers, citizens, NGOs, IOs, etc.) should act.14 Yet

think that political change is driven by material forces, then we are unlikely to see communicative practices of argument and
persuasion as potentially successful sources of change. More than this, though, it is also an issue of epistemology. If we assume that
the proper domain of IR as a social science is the acquisition of empirically verifiable knowledge, then we will struggle to
comprehend, let alone answer, normative questions of how we should act. We will either reduce ought questions to is questions,

Our metatheoretical assumptions thus determine


the macro-orientation of IR towards questions of practice, directly
affecting the fields practical relevance. Secondly, metatheoretical revolutions
license new second-order theoretical and analytical possibilities while
foreclosing others, directly affecting those forms of scholarship widely
considered most practically relevant. The rise of analytical eclecticism illustrates this. As noted above,
or place them off the agenda altogether.15

Katzenstein and Sils call for a pragmatic approach to the study of world politics, one that addresses real-world problematics by
combining insights from diverse research traditions, resonates with the mood of much of the field, especially within the American

Epistemological and ontological debates are widely considered


irresolvable dead ends, grand theorising is unfashionable, and gladiatorial
contests between rival paradigms appear, increasingly, as unimaginative
rituals. Boredom and fatigue are partly responsible for this new mood, but something deeper is at work. Twenty-five years ago,
mainstream.

Sil and Katzensteins call would have fallen on deaf ears; the neo-neo debate that preoccupied the American mainstream occurred
within a metatheoretical consensus, one that combined a neo-positivist epistemology with a rationalist ontology. This singular
metatheoretical framework defined the rules of the game; analytical eclecticism was unimaginable. The Third Debate of the 1980s
and early 1990s destabilised all of this; not because American IR scholars converted in their droves to critical theory or

The antifoundationalist critique of the idea that there is any single measure of
truth did not produce a wave of relativism, but it did generate a
widespread sense that battles on the terrain of epistemology were
unwinnable. Similarly, the Third Debate emphasis on identity politics and
cultural particularity, which later found expression in constructivism, did not vanquish rationalism. It did,
however, establish a more pluralistic, if nevertheless heated, debate about ontology, a
terrain on which many scholars felt more comfortable than that of epistemology. One can plausibly argue, therefore, that the
metatheoretical struggles of the Third Debate created a space for even made
possible the rise of analytical eclecticism and its aversion to metatheoretical
absolutes, a principal benefit of which is said to be greater practical
relevance. Lastly, most of us would agree that for our research to be practically relevant,
it has to be good it has to be the product of sound inquiry, and our conclusions have to be plausible. The pluralists
among us would also agree that different research questions require different methods
of inquiry and strategies of argument. Yet across this diversity there are several practices widely
poststructuralism (far from it), but because metatheoretical absolutism became less and less tenable.

recognised as essential to good research. Among these are clarity of purpose, logical coherence, engagement with alternative
arguments and the provision of good reasons (empirical evidence, corroborating arguments textual interpretations, etc.). Less often

If our epistemological assumptions


affect the questions we ask, then being conscious of these assumptions is
necessary to ensure that we are not fencing off questions of importance,
and that if we are, we can justify our choices. Likewise, if our ontological
assumptions affect how we see the social universe, determining what is in
or outside our field of vision, then reflecting on these assumptions can
prevent us being blind to things that matter. A similar argument applies to our meta-ethical
assumptions. Indeed, if deontology and consequentialism are both meta-ethical
positions, as I suggested earlier, then reflecting on our choice of one or other
position is part and parcel of weighing rival ethical arguments (on issues as diverse
as global poverty and human rights). Finally, our epistemological, ontological and meta-ethical
assumptions are not metatheoretical silos; assumptions we make in one
have a tendency to shape those we make in another. The oft-heard refrain
that if we cant measure it, it doesnt matter is an unfortunate example
of epistemology supervening on ontology, something that metatheoretical
reflexivity can help guard against. In sum, like clarity, coherence, consideration of alternative arguments
noted, however, is the importance of metatheoretical reflexivity.

and the provision of good reasons, metatheoretical reflexivity is part of keeping us honest, making it practically relevant despite its
abstraction.

AT: Science = Truth


Science is not objectiveit reflects particular social practices
that must be understood for scientific conclusions to bear any
meaning
Law 12 Duncan Law, Evidence-Based Belief and Practice, Duncan Laws blog,
7/31/2012, http://duncanlaw.wordpress.com/
Relativism whether it understands itself as anti-science, as a consequence of science, or both is a
common object of critique. The opposing flaw is also a serious one, however: this is the
perspective that grounds sciences authority in an appeal to the way
things are in the world, without seeing how this appeal must itself be
understood as a social practice embedded in a complex system of social
practices. For this approach, in Hegel and Marxs phrase, the process vanishes in the
result: the mechanism by which truth-claims are arrived at is forgotten,
and truth-claims are wielded as if they are the source of sciences social
authority, rather than the result of that authority (as is in fact the case). These
approaches, then, are dogmatic they understand themselves as (and, in most
practical contexts, are) pro-science, but they have an inadequate understanding of
what science is, as a historically-specific set of social practices. Advocates
of this perspective may be able to do science, but they are not able to
adequately justify their findings, without relying on a tacit set of social
norms that their dogmatism overtly denies. Many of the pugnacious contemporary
advocates of science, like Richard Dawkins and Daniel Dennett, belong in this category.

AT: Objectivity k2 Action


Science is not coherent or monolithicobjectivity only
complicates political action
Innerarty 13 Daniel Innerarty, Power and knowledge: The politics of the

knowledge society, European Journal of Social Theory February 2013 16: 3-16,
doi:10.1177/1368431012468801
On the other hand, the relationship between power and knowledge is much more
complex than that assumed by the theory that power is subordinate to
knowledge. At times, in fact, the exact opposite holds: expert knowledge is manipulated
by those in power to justify previously adopted political decisions . In addition,
the world of experts is not generally peaceful or uncontroversial. At times,
political conflicts reproduce the disputes that are taking place in the heart
of the scientific community. Science is rarely able to resolve political
disputes; instead, scientific controversies frequently add fuel to political
disputes. Every expert has a counter-expert, which helps deprive scientific
knowledge of its alleged certainty. Scientific opinion, far from putting an
end to the debate, very often serves to increase the number of
perspectives and consequences that must be taken into consideration . Thus
begins the game with experts on either side, making it clear to the public that, in the case of complex issues with
political and social repercussions,

decisions.

scientific precision in no way ensures rational

AT: Predictions Solve Complexity/Science is SelfCorrecting


Its not a question of having all the factscriticism is key to
coping with uncertainty instead of trying to dream it away
Innerarty 13 Daniel Innerarty, Power and knowledge: The politics of the
knowledge society, European Journal of Social Theory February 2013 16: 3-16,
doi:10.1177/1368431012468801
Jasanoff uses the term technologies of humility (2005: 373) to talk about an institutionalized way of

thinking about the frontiers of human knowledge that which is unknown, uncertain, ambiguous, and uncontrollable

encourages us to consider
the possibility of unforeseen consequences, to make explicit the
normative features that are buried within technical decisions, to recognize
the need for collective learning and multiple points of view. In this context,
rather than the traditional image of a science that produces objective
hard facts, pushing back ignorance and telling politics what to do, we
need a type of science that will cooperate with politics in the management of
uncertainty (Ravetz, 1987: 82). For this reason, we must develop a reflexive culture of
uncertainty that does not perceive non-knowledge as the outer limit of the
yet-to-be investigated (Wehling, 2004: 101), but as an essential part of knowledge
and science. We should not regard that which is not known, uncertain
knowledge, the merely plausible, non-scientific forms of knowledge, and
ignorance as imperfect phenomena but as resources (Bonss, 2003: 49). There are
times when, in the absence of undisputed and unambiguous knowledge,
cognitive strategies must be developed in order to take action within the
bounds of uncertainty. Among the most important types of knowledge are risk assessment,
management, and communication. We must learn to operate in an environment where
the relationship between cause and effect is not clear, but fuzzy and
chaotic.
acknowledging the limits on prediction and control. A similar approach

AT: Big Words Bad/Ivory Tower


Exclusivist critique of social theory for its difficult jargon
neutralizes the political and ideological tropes present in
ordinary language
Zeus Leonardo, Associate Professor of Language and Literacy & Society and
Culture at the UC-Berkeley Graduate School of Education, Critical Social Theory and
Transformative Knowledge: The Functions of Criticism in Quality Education,
Educational Researcher, Vol. 33, No. 6, Aug-Sep 2004, JSTOR
Ideology critique is not merely criticism. As used in common (i.e., uncritical) discourse, criticism is the deployment
of commentaries for political purposes, usually indicative of a Leftist leaning teacher in the popular mind. In this
sense, criticism establishes the superiority of the critic whose impunity the audience often fears. Here critical social
theorists must take some responsibility when their main concern is to become the "ultimate radical" rather than

mainstream audiences often mistake criticism for


political agendas as opposed to engagement, as if only critics have an
agenda. Criticism is (mis)construed as pessimistic, judged as a form of
negativity, and not in the sense that Adorno (1973) once promoted. The teacher-as-critic may be perceived as
promoting dialogue. That said,

aggressive if she teaches the idea that patriarchy is alive and well, as politically incorrect if she cites the white
supremacist origins of the United States, and as homophiliac if she questions the soundness of heterosexual

criticism is at the center of a quality


education that values debate, openness to different ideas, and
commitment to democratic processes. Moreover, pedagogical interactions are
never severed from wider social relations that need to be problematized . In
this sense, criticism is more a search for emancipatory forms of knowledge
and less a contrived condition to honor the critic. Criticism is positioned
here as a central process in promoting a quality education even in the face
of an uneven and unjust world. A language of critique is never simply
about clarity, but is always bound up with a political project. The politics of clarity
families. However, the teacher-as-critic understands that

is particularly important in the reception of CST in education because of its dense theories and descriptions, and

the issue of clarity has been a sore


point in the wider engagement of CST. But clarity is always a question of clarity for
whom and for what? Clarity is too often an issue of conventions and a
critique aiming solely for clarity takes for granted the reader's position
(Giroux, 1995). This does not suggest that critique should aim consciously for
vagueness and obfuscation. For confusion seems opposed to quality. It suggests that critique is
not an issue of either clarity or complexity but both/and. Also, it implicates
clarity as an ideological issue, rather than a merely rhetorical one (Lather,
1996). For example, an uncritical literacy program perpetuates the importance of
clarity over political purpose and denies the fact that people's tastes and
dispositions toward language are socially motivated. As a case in point, language
learning frequently intersects issues of race, forming what Hopson (2003) calls the
"the problem of the language line." Hopson combines Bourdieu's concept of linguistic capital with
Du Bois' thoughts on the "color line" and finds that language learning is never just about
induction into mainstream schooling but a way to perpetuate linguistic
racism, in this case through the hegemony of English (see also Macedo, Dendrinos, &
Gounari, 2003). Criticism launched against the apparently muddy descriptions
of CST tends to valorize ordinary language. It is not uncommon that
mainstream educators charge that critical educational language is "elitist"
or "exclusivist." Its highly academic discourse is not only hard to understand, it seems to demand much
previous knowledge from its readers. Though this particular criticism helps point to the important project of
therefore warrants some critical attention. Over the years,

misses the mark because quality education is


proportional to the depth of one's analysis, part of which is the
engagement with theoretical discourse. It assumes problematically that
ordinary language is sufficient and non-ideological (Gouldner, 1976; Aoki, 2000). The
argument valorizes common language as transparent when compared to
the supposed opacity of critical language. In fact, there is much in ordinary
language that leaves one searching for a better mode of critique in terms
of providing educators, teachers, and administrators discourses for a deeper
engagement of school processes and hence a quality experience. It is for this
widening the interest in CST, it also

reason that Said prefers the phrase "historical experience" because it is not esoteric (therefore accessible) but not
without its theoretical moorings that a critical social theorist like Said (2000) proceeds to unpack. Likewise, CST in
education works to build a language of depth hermeneutics and as such maintains its critical edge while at the
same time fashioning it out of people's concrete lives or lived experiences.

AT Agonism

AT: Nietzschean Agonism 2AC


You have Nietzsche on agonism backwards affirming life
means meeting us on the battlefield of argument instead of
hiding behind the safe and familiar walls of framework and
priding yourself for your ascetic ability to work within limits
Acampora 2 Christa Davis Acampora, professor of philosophy at Hunter College
of CUNY, Of Dangerous Games and Dastardly Deeds, International Studies in
Philosophy, vol. 34, no. 3, Fall 2002

II. Dastardly Deeds The so-called "Good Eris," described in "Homer's Contest," supposedly allowed the unavoidable
urge to strive for preeminence to find expression in perpetual competition in ancient Greek culture. In On the

Nietzsche seeks to critique Christianity for advocating a kind of


altruism, or selflessness, that is essentially self-destructive, and for
perverting the urge to struggle by transforming it into a desire for
annihilation. Read in light of "Homer's Contest," Nietzsche's Genealogy enables us to
better grasp his conception of the value of contest as a possible arena for
the revaluation of values, and it advances an understanding of the
distinctions Nietzsche draws between creative and destructive forms of
contest and modes of competing within them. Nietzsche's On the Genealogy of
Morals, a Streitschrifta polemic, a writing that aims to provoke a certain kind of fighting portrays a
battle between "the two opposing values 'good and bad,' 'good and evil'."
Nietzsche depicts slavish morality as that which condemns as evil what
perpetuates the agonnamely, self-interest, jealousy, and the desire to legislate values but
rather than killing off the desire to struggle, slavish morality manipulates
and redirects it. Prevention of struggle is considered by Nietzsche to be hostile to
life: an "order thought of as sovereign and universal, not as a means in
the struggle between power-complexes but as a means of preventing all
struggle in general... would be a principle hostile to life, an agent of the
dissolution and destruction of man, an attempt to assassinate the future
of man, a sign of weariness, a secret path to nothingness" (GM II:11). "The
'evolution' of a thing, a custom, an organ is [...] a succession of [...] more or less mutually
independent processes of subduing, plus the resistances they encounter,
the attempts at transformation for the purpose of defense and reaction,
and the results of successful counteractions"(GM II:12). For Nietzsche, human beings, like
Genealogy of Morals,

nations, acquire their identity in their histories of struggles, accomplishments, and moments of resistance. The
complete cessation of strife, for Nietzsche, robs a being of its activity, of its life. In the second essay of the
Genealogy, Nietzsche identifies the notion of conscience, which demands a kind of self-mortification, as an example
of the kind of contest slavish morality seeks: "Hostility, cruelty, joy in persecuting, in attacking, in change, in
destructionall this turned against the possessors of such instinct: that is the origin of the 'bad conscience'" (GM
II:16). Denied all enemies and resistances, finding nothing and no one with whom to struggle except himself, the
man of bad conscience: impatiently lacerated, persecuted, gnawed at, assaulted, and maltreated himself; this
animal that rubbed itself raw against the bars of its cage as one tried to 'tame' it; this deprived creature... had to
turn himself into an adventure, a torture chamber, an uncertain and dangerous wilderness this fool, this yearning
and desperate prisoner became the inventor of the 'bad conscience.' But thus began the gravest and uncanniest
illness... a declaration of war against the old instincts upon which his strength, joy, and terribleness had reached
hitherto (GM II:16). Bad conscience functions in slavish morality as a means of self-flagellation, as a way to vent the
desire to hurt others once external expressions of opposition are inhibited and forbidden. "Guilt before God: this
thought becomes an instrument of torture to him" (GM II:22). In that case, self-worth depends upon the ability to
injure and harm oneself, to apply the payment of selfmaltreatment to one's irreconcilable account with God. It is the
effort expended in one's attempt to make the impossible repayment that determines one's worth. xi The genuine
struggle, that which truly determines value for the ascetic ideal is one in which one destructively opposes oneself

Slavish morality is still driven by


contest, but the mode of this contest is destructive. It mistakes selfone's value increases as one succeeds in annihilating oneself.

inflicted suffering as a sign of strength. The ascetic ideal celebrates cruelty and
tortureit revels in and sanctifies its own pain. It is a discord that wants to be discordant,
that enjoys itself in this suffering and even grows more self-confident and
triumphant the more its own presupposition, its physiological capacity for
life decreases. 'Triumph in the ultimate agony': the ascetic ideal has always fought under this hyperbolic
sign; in this enigma of seduction, in this image of torment and delight, it recognized its brightest light, its salvation,
its ultimate victory (GM III:28). Slavish morality, particularly in the form of Pauline Christianity, redirects the
competitive drive and whips into submission all outward expressions of strife by cultivating the desire to be "good"
xii in which case being good amounts abandoning, as Nietzsche portrays it, both the structure of the contests he
admired in "Homer's Contest" and the productive ways of competing within them. It does not merely redirect the
goal of the contest (e.g., struggling for the glory of Christ rather than competing for the glory of Athens), rather how
one competes well is also transformed (e.g., the "good fight" is conceived as tapping divine power to destroy
worldly strongholds xiii rather than excelling them). In other words, the ethos of contest, the ethos of the agon is

Moralities effect contests in two ways: 1) they


articulate a structure through which the meaning of human being (e.g.,
excellence, goodness, etc.) can be created and meted out, and 2) they simultaneously
cultivate a commitment to a certain way of competing within those
structures. By cultivating not only a desire to win but a desire to compete
well (which includes respect for one's competitor and the institutions that sets forth the terms of the
engagement), xv we can establish a culture capable of deriving our standards of
excellence internally and of renewing and revaluing those standards
according to changes in needs and interests of our communities. This is the
transformed in slavish morality. Xiv III. Dangerous Games

legacy that Nietzsche strives to articulate in his "Homer's Contest," one that he intends his so-called "new nobility"

If the life of slavish morality is characterized by actions of


annihilation and cruelty, Nietzsche's alternative form of valuation is
marked by its activity of surmounting what opposes, of overcoming
opposition by rising above (erheben) what resists, of striving continually to
rise above the form of life it has lived. As a form of spiritualized striving, self-overcoming,
to claim.

must, like Christian agony, be self-directed; its aim is primarily resistance to and within oneself, but the agonythat
is, the structure of that kind of painful strugglediffers both in how it orients its opposition and in how it pursues its

Self-overcoming does not aim at self-destruction but rather at selfexhaustion and self-surpassing. It strives not for annihilation but for
transformation, and the method of doing so is the one most productive in
the external contests of the ancient Greeks: the act of rising above. Selfovercoming asks us to seek hostility and enmity as effective means for
summoning our powers of development. Others who pose as resistances,
who challenge and test our strength, are to be earnestly sought and
revered. That kind of reverence, Nietzsche claims, is what makes possible genuine relationships that
enhance our lives. Such admiration and cultivation of opposition serve as "a bridge to love" (GM I:10)
because they present a person with the opportunity to actively distinguish
himself, to experience the joy and satisfaction that comes with what Nietzsche describes as
"becoming what one is." xvi This, Nietzsche suggests, is what makes life worth livingit is what
goals.

permits us to realize a certain human freedom to be active participants in shaping our own lives. xvii

Particularly true in the context of topicality which requires


translation of an arbitrary set of signifiers into iron pillars of
Truth this will to mastery over the chaotic meaning of the
resolution is true reactivity
Spivak 76 Gayatri Chakravorty Spivak, Translators Preface to Of Grammatology,
Baltimore: Johns Hopkins University Press, 1976, p. xxii-xxiv
It should by now be clear that Nietzsche's "suspicion of the value of truth . . . of
meaning and of being, of `meaning of being' " of the "concept of . . . the primary
signified," is intimately shared by Derrida. The other items on the two lists can be brought

under one head: philosophical dis-course as formal, rhetorical, figurative discourse, a something to be deciphered.
The end of this Preface will make clear how deeply Derrida is committed to such a notion. Here I shall comment on

Nietzsche
described metaphor as the originary process of what the intellect presents
as "truth." "The intellect, as a means for the preservation of the
individual, develops its chief power in dissimulation." 20 "A nerve-stimulus,
first transcribed [bertragen] into an image [Bild]! First metaphor! The image
again copied into a sound! Second metaphor! And each time he [the creator
of language] leaps completely out of one sphere right into the midst of an
entirely different one." (NW III. ii. 373, TF 178) In its simplest outline, Nietzsche's definition of
the implications of "the decipherment of figurative discourse" in Nietzsche. As early as 1873,

metaphor seems to be the establishing of an identity between dissimilar things. Nietzsche's phrase is "Gleich
machen" (make equal), calling to mind the German word "Gleichnis"image, simile, similitude, comparison,
allegory, parablean unmistakable pointer to figurative practice in general .

"Every idea originates


through equating the unequal." (NW III. ii. 374, TF 179) "What, therefore, is truth?
A mobile army of metaphors , metonymies, anthropomorphisms; . . . truths are illusions
of which one has forgotten that they are illusions, . . . coins which have
their obverse effaced and now are no longer of account as coins but
merely as metal." (NW III. ii. 374-75, TF 18o) I hold on here to the notions of a process of figuration and a
process of forgetfulness. In this early text, Nietzsche describes the figurative drive as "that im-pulse towards the
formation of metaphors, that fundamental impulse of man, which we cannot reason away for one momentfor
thereby we should reason away man himself. . . . (NW III. ii. 381, TF 188) Later he will give this drive the name "will
to power." Our so-called will to truth is a will to power because "the so-called drive for knowledge can be traced
back to a drive to appropriate and conquer." 21 Nietzsche's sense of the inevitable forcing of the issue, of exercising
power, comes through in his italics: " `Thinking' in primitive conditions (preorganic) is the crystallization of forms. . .
. In our thought, the essential feature is fitting new mate-rial into old schemas, . . . making equal what is new." 22

The human being has nothing more to go on than a collection of nerve


stimuli. And, because he or she must be secure in the knowledge of, and
therefore power over, the "world" (inside or outside), the nerve stimuli are explained
and described through the categories of figuration that masquerade as the
categories of "truth." These explanations and descriptions are
"interpretations" and reflect a human inability to tolerate undescribed
chaos "that the collective character [Gesamtcharakter] of the world . . . is in all eternity chaos in the sense

not of a lack of necessity but of a lack of order, arrangement, form, beauty, wisdom, and whatever other names
there are for our aesthetic anthropomorphisms [human weaknessesMenschlichkeiten]."23 As Nietzsche suggests,

this need for power through anthropomorphic defining compels humanity


to create an unending proliferation of interpretations whose only "origin,"
that shudder in the nerve strings, being a direct sign of nothing, leads to
no primary signified. As Derrida writes, Nietzsche provides an "entire thematics
of active interpretations, which substitutes an incessant deciphering for
the disclosure of truth as a presentation of the thing itself." (MP 19, SP 149)
Interpretation is "the introduction of meaning" (or "deception through meaning"
Sinnhineinlegen), a making-sign that is a making-figure, for there is, in this
thought, no possibility of a literal, true, self-identical mean-ing.
Identification (Gleich-machen) constitutes the act of figuration. Therefore, "nothing
is ever comprehended, but rather designated and distorted...." This extends, of

course, to the identity between an act (effect) and its purpose (cause) : "Every single time something is done with a
purpose in view, something fundamentally different and other occurs." (WM H. 59, 130; WP 301, 351) The will to
power is a process of "incessant deciphering"figurating, interpreting, sign-ifying through ap-parent identification.
Thus, even supposing that an act could be isolated within its outlines, to gauge the relationship between it and its

the critical glance must reverse (necessarily nonidenticallv) this


decipherment, follow the "askew path," read the act in its textuality . In this
"originating" consciousness,

important respect, "without him [Nietzsche] the 'question' of the text would never have erupted, at least in the

Nietzsche reads the history of


morality as a text. He interprets the successive meanings of systems of
precise sense that it has taken today."24 In The Genealogy of Morals,

morality. "Pur-poses and utilities are only signs that a will to power has
become master of something less powerful and has in turn imprinted the
meaning of a function upon it [ihm von sich aus den Sinn einer Funktion auf geprgt hat; this image
of Au f prgungimprinting`figuration' in yet another sense, is most important in Nietzsche, and constantly

the entire history of a `thing', an organ, a custom can in


this way be a continuous sign-chain of ever new interpretations and makeshift excuses [Zurechtmachungen] whose causes do not even have to be related to one another in a purely
recurs in this particular context]; and

chance fashion."25 "All concepts in which an entire process is semiotically telescoped [Zusammenfasst] elude
definition." (NW, VI. ii. 333, GM 8o) Derrida would, of course, suspend the entire notion of semiosis, put the sign
under erasure. It is possible to read such a suspension into Nietzsche's "continuous sign-chains," without origin and
end in "truth." And thus it is possible to discover an affinity between Derrida's practice in Of Grammatology and
Nietzsche's interpretation of value systems as infinite textuality; and to see in Derrida's decipherment of the
negative valuation of writing within the speech-writing hierarchy the mark of a Nietzschean "genealogy."

Critical Agonism Good


Agonism requires contestation of a sufficiently reflexive
character that it allows even itself to be put to question; to
stabilize debate along the lines of a static resolution is to put
debates agonistic potential to death
Siemens 1 Herman W. Siemens, Department of Philosophy at Nijmegen

University in The Netherlands, Nietzshces agon with ressentiment: towards a


therapeutic reading of critical transvaluation, Continental Philosophy Review 34:
69-93, 2001, SpringerLink
Nietzschean transvaluation is devoted to contesting and overcoming
prevailing values in the name of life, its affirmation and elevation. As an
agonal contest of values, transvaluative discourse challenges a given
value or ideal by staging a confrontation with a representative persona or
type, whose various attitudes and postures are then interrogated and
evaluated from a standpoint in life as the highest value. The agon has important
consequences for the way we understand Nietzsches textual confrontations, which can be used to introduce a

the agon involves a symmetrical organisation


or economy of power, presupposing a plurality of more-or-less equal
antagonistic forces. 13 Agonal discourse is therefore contingent on the
participation of a plurality of forces in a symmetrical contestation of
values: transvaluation only occurs where we are drawn into critical
contests, as an agonal community of readers who consent to the rules
of play. Under these conditions, deference to Nietzsche or any single force is
ruled out. Nietzsches judgements do of course claim authority, but not
the incontrovertible authority of truth-claims delivered by a great master,
healer or priest. They serve rather to open play, to provoke dispute and
draw us into controversy; like Zarathustra, Nietzsche would sooner have hated
friends than command belief (EH pref. 4). Agonal authorship throws its own
authority in the balance, to be won by purely human means of consensus:
judgements and counter-values, together with the very standards of
evaluation or judgement, are opened to contestation by a collective
readership which would respond to the challenge it issues. In this light, agonal
viable therapeutic reading. In the first place,

hermeneutics can accommodate at least one of Nietzsches counter-therapeutic impulses: the rejection of

Conclusive
victory for any antagonist spells the death of the agon: since the agon
precludes both conclusive defeat (destruction) and conclusive victory, it is
repeatable and inconclusive in its very mode of being . As a consequence, the
agon gives an open-ended, inconclusive orientation to transvaluative
discourse. Despite its popular image, Nietzschean critique does not aim to destroy its
opponents (life-negating values or attitudes like ressentiment) and
assert a single-handed victory (conclusive counter-values) over them. Instead, it
serves to open and re-open the question of victory. 14 What would constitute the
asymmetrical (saviour/priest-sinner; master-disciple) relationships voiced by Zarathustra.

overcoming of life-negating values? What would be an affirmative practice beyond ressentiment? In this light,
agonal hermeneutics addresses the most serious threat to a therapeutic reading: the redemptive desire to destroy

If it is declining forms of life that dream of annihilating


antagonistic forces for the sake of peace, then the interests of
ascending life, by contrast, require the empowerment of the antagonist, for
the sake of continued conflict and growth. Nietzsches philosophy must
Christian-Platonic values.
[Vernichtung]

therefore resist the lure of finality and the expedient of destroying its
opponents. This does not exclude conflict altogether. The interests of
growing, struggling life require that Nietzsches philosophy practise
conflict or struggle in a form that (a) empowers its opponents, 15 and (b)
remains open-ended or inconclusive; that is, it must practice agonal conflict.

AT: You Destroy Debate


Debate is continually being destroyed; thats precisely the
nature of agonistic community that we should affirm.
Siemens 1 Herman W. Siemens, Department of Philosophy at Nijmegen

University in The Netherlands, Nietzsches agon with ressentiment: towards a


therapeutic reading of critical transvaluation, Continental Philosophy Review 34:
69-93, 2001, SpringerLink
If understood correctly, the open-ended, dynamic qualities of the agon also
address the problem of closure at its most intractable: the demand that
therapeutic discourse be non-directional or anti-teleological, in the
interests of ascending life. Repeatability in the agon cannot be properly
understood if we restrict ourselves to a historical perspective, or the
perspective of the antagonists. At issue is not whether the agon is in fact
repeated, and the kinds of institutional and financial infrastructure this
requires. These historical/empirical questions presuppose that the agon is to be repeated, and this is a feature
intrinsic to the agon as a dynamic ordering of forces, a matter of its temporal character as a festival and a form of

play cannot be adequately understood from the


players perspectives, because it has its being independently of their
consciousness, attitudes and intentions: the mode of being of play is not
such that there must be a subject who takes up a playing attitude so that
the game can be played. Rather, the most original sense of play is the medial
sense. 16 In this sense, play acquires a structure of repetition that is radically
impersonal and anti-teleological. Whatever the players intentions, their outcome is
determined in the space of play or confrontation, so that the real subject
of play is not the player, but play itself which holds the player in thrall (p.
106). From this perspective, the dynamics of play are freed from the players
intentions, goals and efforts, which are themselves played out within a toand-fro movement detached from any telos: the movement which is play,
has no goal which brings it to an end; rather it renews itself in constant
repetition (p. 103). This thought is fleshed out by Gadamer with reference to the puzzling temporality of
(periodic) festivals. 17 The festival cannot be properly grasped from the usual
perspective in successive time, as a historical event that was originally so
and then came to be repeated with small variations at periodic intervals .
Rather, repetition or return is intrinsic to festivals including agonal festivals in their
character as celebration. Since it belongs to the establishment of a
festival, at its very origins, that it should be regularly celebrated, the
festival is something that only is insofar as it is always different [. . .] It has
its being only in becoming and recurring (p. 120). In this light, the open-ended
repeatability of agonal discourse is not contingent on the self-restraint of
antagonists able to hold back from destruction or absolute victory. Contestants
cannot be relied on to avoid excess in the agon which, by its very nature, allows the temptation
play. As Gadamer has argued,

of hubris to compete with the warning of self-restraint with uncertain results. This goes for Nietzsche as well, who

antagonists must be clearly


distinguished from the agon itself, as the subject of play in the medial
sense. Whatever their attitudes or intentions, they are, as agonal players,
subject to the to-and-fro dynamics of empowerment-disempowerment, an
inconclusive, repeatable movement detached from any telos. If , as I suggest,
the agon gives the temporal character of play and celebration to
is notoriously unrestrained at times. According to Gadamer, however, the

Nietzsches textual confrontations, then we can say: agonal discourse is a


radically impersonal, non-directional and repeatable medium of thought;
something that only is insofar as it is becoming. Individual teleologies are
embedded in the anti-teleological medium of agonal exchange to which
they give themselves; any bids for power, any attempts at closure are
checked or undone by the vicissitudes of empowerment-disempowerment
to which they are subject. Agonal hermeneutics thus ensures that
Nietzsches therapeutic interests remain non-directional and open- ended,
in line with the interests of ascending life, despite the temptations to
closure that haunt his project.

AT: Positivism/Objectivity
Agonism requires that the game isnt decided before it begins
frameworks reliant on absolute objectivity negate this by
recasting the encounter as a question of right
Siemens 1 Herman W. Siemens, Department of Philosophy at Nijmegen
University in The Netherlands, Nietzshces agon with ressentiment: towards a
therapeutic reading of critical transvaluation, Continental Philosophy Review 34:
69-93, 2001, SpringerLink

But there is, it seems, a difficulty here. For it is hard to see how agonal discourse, if non-directional, can promote
the interests of ascending life. How can a non-directional medium be in any sense orientated towards health? The
answer I propose involves the feint of writing, that is, the emphatically fictive style of Nietzsches agonal
confrontations. It was noted earlier how well the agon exemplifies the notion of ascending life advocated in GS 370.

agonal discourse enacts the highest form of


life for Nietzsche (growth, fertility, conflict, excess). The agonal dynamic
regulating his discourse serves to supplement the discursive critique of
pathological, life-negating regimes with a performative challenge that
anticipates or pre-figures the therapeutic telos of health a productive
and affirmative form of life. 18 The notion of fiction is important for two reasons. First,
because it involves a particular vision, a possible form of life or health
amongst others, not a normative concept of health enjoined upon all as a
binding universal law or telos. The distance between teleology and fiction
is measured by the difference between enclosing the horizon of the future,
and playing with an open horizon. The agonal feint thus orients
transvaluation towards health without subsuming it under goals or
directives that would in fact promote the interests of declining life: the
anti-teleology of fiction joins the anti-teleology of play. Fiction is also
important in its performative aspect as the agonal dynamic of mutual
empowerment-disempowerment enacted in Nietzsches texts. This agonal dynamic
throws valuable light on certain features of Nietzsches thinking that
resist discursive understanding; it also opens up an energetic dimension
to Nietzsches texts essential to their therapeutic potential. A recurrent and highly
Building on this observation, I propose that

problematic feature of Nietzsches critical and interpretative style is a characteristic movement of saying and
unsaying (Blondel). This can take different forms: as an alternation of appropriation and alienation, 19 of
dominating and freeing the other in turn, or Nietzsches tendency to limit his negations of the other through
subsequent affirmation to name a few. Common to all is a double-movement of Absolutsetzung and NichtAbsolutsetzung 20 whereby Nietzsche contests a position and then retracts his contention, or opposes a claim only
to undo his counter-claim. From a discursive point of view, all this is hard to make sense of, or simply incoherent.

The agon, as we
have seen, precludes destruction of the opponent in favor of a practice of
limited aggression (mutual disempowerment) predicated on mutual empowerment.
From a dynamic perspective in agonal contention, however, it begins to make sense.

If, as I am suggest- ing, Nietzsches textual confrontations are regulated by an agonal regime, then they are bound
to unfold through a dynamic of mutual empowerment-disempowerment. Within this dynamic, saying and
unsaying constitutes a coherent practice of limited aggression. At stake here is how we read Nietzsche: instead of
isolating his judgements or interpretations from one another and identifying them with contradictory positions, we
need to place them within an agonal play of forces that implicates us as readers, not just his chosen adversary, in
a collective contestation of values.

AT Deliberative Democracy

Deliberative Democracy Bad


Deliberative democracy solves none of our criticismits
presumption of a neutral public sphere erases the way the
terms of the debate are constituted by contingent and nonnecessary power relations
Mouffe 2k Chantal Mouffe, professor in the Department of Politics and

International Relations at the University of Westminster, Deliberative Democracy or


Agonistic Pluralism, Reihe Politikwissenschaft Political Science Series 72, December
2000, probably found on google scholar
Besides putting the emphasis on practices and language games, an alternative to the rationalist framework also
requires coming to terms with the fact that power is constitutive of social relations. One of the shortcomings of the

by postulating the availability of a public sphere where


power would have been eliminated and where a rational consensus could
be realized, this model of democratic politics is unable to acknowledge the
dimension of antagonism that the pluralism of values entails and its ineradicable
character. This is why it is bound to miss the specificity of the political, which
it can only envisage as a specific domain of morality . Deliberative democracy provides
a very good illustration of what Carl Schmitt had said about liberal thought: In a very systematic
fashion liberal thought evades or ignores state and politics and moves instead
in a typical always recurring polarity of two heterogeneous sphere, namely ethics and
economics.30 Indeed, to the aggregative model, inspired by economics, the only alternative deliberative
deliberative approach is that,

democrats can put forward is one that collapses politics into ethics. In order to remedy this serious deficiency, we
need a democratic model able to grasp the nature of the political. This requires developing an approach, which
places the question of power and antagonism at its very center. It is such an approach that I want to advocate and
whose theoretical bases have been delineated in Hegemony and Socialist Strategy.31 The central thesis of the book

social objectivity is constituted through acts of power. This implies


that any social objectivity is ultimately political and that it has to show the
traces of exclusion, which governs its constitution. This point of
convergence or rather mutual collapse between objectivity and power
is what we meant by hegemony. This way of posing the problem indicates that power should
not be conceived as an external relation taking place between two preconstituted identities, but rather as constituting the identities
themselves. Since any political order is the expression of a hegemony, of a
specific pattern of power relations, political practice cannot be envisaged
in simply representing the interests of pre-constituted identities, but in
constituting those identities themselves in a precarious and always
vulnerable terrain. To assert the hegemonic nature of any kind of social order is to operate a
displacement of the traditional relation between democracy and power. According to the
deliberative approach, the more democratic a society is, the less power
would be constitutive of social relations. But if we accept that relations of
power are constitutive of the social, then the main question for
democratic politics is not how to eliminate power but how to constitute
forms of power more compatible with democratic values. Coming to terms
with the constitutive nature of power implies relinquishing the ideal of a
democratic society as the realization of a perfect harmony or
transparency. The democratic character of a society can only be based on
the fact that no limited social actor can attribute to herself the
representation of the totality and claim to have the mastery of the
foundation. Democracy requires, therefore, that the purely constructed nature
is that

of social relations finds its complement in the purely pragmatic grounds of


the claims to power legitimacy. This implies that there is no unbridgeable gap between power and
legitimacy not obviously in the sense that all power is automatically legitimate, but in the sense that: a) if any
power has been able to impose itself, it is because it has been recognized
as legitimate in some quarters; and b) if legitimacy is not based on a
aprioristic ground, it is because it is based on some form of successful
power. This link between legitimacy and power and the hegemonic order
that this entails is precisely what the deliberative approach forecloses by
positing the possibility of a type of rational argumentation where power
has been eliminated and where legitimacy is grounded on pure rationality.

The drive for consensus reproduces violent ideology and shuts


down substantive debate, making each a rehearsal of what
came beforeinnovation and creativity is possible not only
when debate occurs over particular issues, but the conceptual
frameworks undergirding them
Mouffe 2k Chantal Mouffe, professor in the Department of Politics and

International Relations at the University of Westminster, Deliberative Democracy or


Agonistic Pluralism, Reihe Politikwissenschaft Political Science Series 72, December
2000, probably found on google scholar
One of the key thesis of agonistic pluralism is that, far from jeopardizing democracy,
agonistic confrontation is in fact its very condition of existence. Modern
democracys specificity lies in the recognition and legitimation of conflict
and the refusal to suppress it by imposing an authoritarian order.
Breaking with the symbolic representation of society as an organic body
which was characteristic of the holist mode of social organization a
democratic society acknowledges the pluralism of values, the disenchantment of
the world diagnosed by Max Weber and the unavoidable conflicts that it entails. I agree
with those who affirm that a pluralist democracy demands a certain amount of consensus and that it requires

since those ethico-political


principles can only exist through many different and conflicting
interpretations, such a consensus is bound to be a conflictual
consensus. This is indeed the privileged terrain of agonistic confrontation
among adversaries. Ideally such a confrontation should be staged around the diverse conceptions of
allegiance to the values, which constitute its ethico-political principles. But

citizenship, which correspond to the different interpretations of the ethico-political principles: liberal-conservative,

Each of them proposes its own


interpretation of the common good, and tries to implement a different
form of hegemony. To foster allegiance to its institutions, a democratic
system requires the availability of those contending forms of citizenship
identification. They provide the terrain in which passions can be mobilized
around democratic objectives and antagonism transformed into agonism.
A well functioning democracy calls for a vibrant clash of democratic
political positions. If this is missing there is the danger that this
democratic confrontation will be replaced by a confrontation among other
forms of collective identification, as it is the case with identity politics. Too much
emphasis on consensus and the refusal of confrontation lead to apathy
and disaffection with political participation. Worse still, the result can be the
crystallization of collective passions around issues, which cannot be
managed by the democratic process and an explosion of antagonisms that
can tear up the very basis of civility. It is for that reason that the ideal of a
social-democratic, neo-liberal, radical-democratic, etc.

pluralist democracy cannot be to reach a rational consensus in the public


sphere. Such a consensus cannot exist. We have to accept that every
consensus exists as a temporary result of a provisional hegemony, as a
stabilization of power, and that it always entails some form of exclusion.
The idea that power could be dissolved through a rational debate and that
legitimacy could be based on pure rationality are illusions, which can
endanger democratic institutions. What the deliberative democracy model
is denying is the dimension of undecidability and the ineradicability of
antagonism, which are constitutive of the political. By postulating the
availability of a non exclusive public sphere of deliberation where a
rational consensus could be obtained, they negate the inherently
conflictual nature of modern pluralism. They are unable to recognize that
bringing a deliberation to a close always results from a decision which
excludes other possibilities and for which one should never refuse to bear
responsibility by invoking the commands of general rules or principles . This
is why a perspective like agonistic pluralism which reveals the impossibility of establishing a consensus without
exclusion is of fundamental importance for democratic politics. By warning us again of the illusion that a fully

To make
room for dissent and to foster the institutions in which it can be
manifested is vital for a pluralist democracy and one should abandon the
very idea that there could ever be a time in which it would cease to be
necessary because the society is now well ordered. An agonistic
approach acknowledges the real nature of its frontiers and the forms of
exclusion that they entail, instead of trying to disguise them under the
veil of rationality or morality.
achieved democracy could ever be instantiated, it forces us to keep the democratic contestation alive.

Deliberative democracy is incapable of resolving


incommensurate social concerns
Mouffe 2k Chantal Mouffe, The Democratic Paradox, Verso: New York, 2000, p.
45-46
the impossibility of establishing a
rational consensus without exclusion. This raises several problems for the model
of democratic politics, which has been receiving quite a lot of attention recently under the name 'deliberative
democracy'. No doubt, the aim of the theorists who advocate the different versions of such a model is
One of the implications of the argument presented above is

commendable. Against the interest-based conception of democracy, inspired by economics and sceptical about the
virtues of political participation, they want to introduce questions of morality and justice into politics, and envisage

by proposing to view reason and rational


argumentation, rather than interest and aggregation of preferences, as
the central issue of politics, they simply replace the economic model with
a moral one which - albeit in a different way - also misses the specificity of the
political. In their attempt to overcome the limitations of interest-group pluralism, deliberative
democrats provide a telling illustration of Schmitt's point that 'In a very
systematic fashion liberal thought evades or ignores state and politics and
moves instead in a typical, always recurring polarity of two heterogeneous
spheres, namely ethics and economics, intellect and trade, education and
property.'13
democratic citizenship in a different way. However,

Deliberations presumption of a neutral discursive sphere


conceals the work of power in legitimating political identity
and the public/private distinctionwe should instead affirm the
necessity of violence within politics
Mouffe 2k Chantal Mouffe, The Democratic Paradox, Verso: New York, 2000, p.
98-101
an alternative to the rationalist
framework also requires coming to terms with the fact that power is
constitutive of social relations. One of the shortcomings of the
deliberative approach is that, by postulating the availability of a public
sphere where power would have been eliminated and where a rational
consensus could be realized, this model of democratic politics is unable to
acknowledge the dimension of antagonism that the pluralism of values
entails and its ineradicable character. This is why it is bound to miss the
specificity of the political which it can only envisage as a specific domain
of morality. Deliberative democracy provides a very good illustration of what Carl Schmitt had said about
Besides putting the emphasis on practices and language-games,

liberal thought: 'In a very systematic fashion liberal thought evades or ignores state and politics and moves instead
in a typical always recurring polarity of two heterogeneous spheres, namely ethics and economics.'ll Indeed, to the
aggregative model, inspired by economics, the only alternative deliberative democrats can oppose is one that

we need a democratic model


able to grasp the nature of the political. This requires developing an
approach which places the question of power and antagonism at its very
centre. It is such an approach that I want to advocate and whose theoretical bases have been delineated in
Hegemony and Socialist Strategy.29 The central thesis of the book is that social objectivity is
constituted through acts of power. This implies that any social objectivity
is ultimately political and that it has to show the traces of exclusion which
governs its constitution. This point of convergence - or rather mutual collapse between objectivity and power is what we meant by 'hegemony' . This way of
posing the problem indicates that power should not be conceived as an external
relation taking place between two preconstituted identities, but rather as
constituting the identities themselves. Since any political order is the
expression of a hegemony, of a specific pattern of power relations, political practice
cannot be envisaged as simply representing the interests of
preconstituted identities, but as constituting those identities themselves
in a precarious and always vulnerable terrain. To assert the hegemonic nature of any kind
collapses politics into ethics. In order to remedy this serious deficiency,

of social order is to operate a displacement of the traditional relation between democracy and power.

According to the deliberative approach, the more democratic a society is,


the less power would be constitutive of social relations. But if we accept
that relations of power are constitutive of the social, then the main
question for democratic politics is not how to eliminate power but how to
constitute forms of power more compatible with democratic values.
Coming to terms with the constitutive nature of power implies
relinquishing the ideal of a democratic society as the realization of a
perfect harmony or transparency. The democratic character of a society
can only be given by the fact that no limited social actor can attribute to
herself or himself the representation of the totality and claim to have the
'mastery' of the foundation. Democracy requires, therefore, that the purely
constructed nature of social relations finds its complement in the purely
pragmatic grounds of the claims to power legitimacy . This implies that there is
no unbridgeable gap between power and legitimacy - not obviously in the sense that

if any power has been able to


impose itself, it is because it has been recognized as legitimate in some
quarters: and (b) if legitimacy is not based in an aprioristic ground, it is
because it is based in some form of successful power. This link between
legitimacy and power and the hegemonic ordering that this entails is
precisely what the deliberative approach forecloses by positing the
possibility of a type of rational argumentation where power has been
eliminated and where legitimacy is grounded on pure rationality .
all power is automatically legitimate, but in the sense that: (a)

Deliverative democracys dream of a politics without


disagreement requires a social homogeneity which closes the
space of the political and excludes those who cannot fit within
the regulative ideal of rationality
Mouffe 2k Chantal Mouffe, The Democratic Paradox, Verso: New York, 2000, p.
104-105
What the deliberative-democracy model is denying is the dimension of
undecidability and the ineradicability of antagonism which are constitutive
of the political. By postulating the availability of a non-exclusive public
sphere of deliberation where a rational consensus could obtain, they
negate the inherently conflictual nature of modern pluralism. They are
unable to recognize that bringing a deliberation to a close always results
from a decision which excludes other possibilities and for which one
should never refuse to bear responsibility by invoking the commands of
general rules or principles. This is why a perspective like 'agonistic pluralism',
which reveals the impossibility of establishing a consensus without
exclusion, is of fundamental importance for democratic politics. By
warning us against the illusion that a fully achieved democracy could ever
be instantiated, it forces us to keep the democratic contestation alive. To
make room for dissent and to foster the institutions in which it can be
manifested is viral for a pluralist democracy, and one should abandon the
very idea that there could ever be a time in which it would cease to be
necessary because the society is now 'well-ordered'. An 'agonistic'
approach acknowledges the real nature of its frontiers and the forms of
exclusion that they entail, instead of trying to disguise them under the
veil of rationality or morality. Coming to terms with the hegemonic nature
of social relations and identities, it can contribute to subverting the everpresent temptation existing in democratic societies to naturalize its
frontiers and essentialize its identities. For this reason it is much more receptive
than the deliberative model to the multiplicity of voices that contemporary
pluralist societies encompass and to the complexity of their power
structure.

Consensus Bad
Denying the paradox between unity and difference produces
consensuses of imperialisminstead we should negotiate this
paradox deconstructively
Mouffe 2k Chantal Mouffe, The Democratic Paradox, Verso: New York, 2000, p.
92-93

The second issue is another question that concerns the relation between private autonomy and political autonomy.
As we have seen, both authors aim at reconciling the 'liberties of the ancients' with the 'liberties of the moderns'
and they argue that the two types of autonomy necessarily go together. However, Habermas considers that only his
approach manages to establish the co-originality of individual rights and democratic participation. He affirms that
Rawls subordinates democratic sovereignty to liberal rights because he envisages public autonomy as a means to
authorize private autonomy. But as Charles Larmore has pointed out, Habermas, for his part, privileges the
democratic aspect, since he asserts that the importance of individual rights lies in their making democratic selfgovernment possible.19 So we have to conclude that, in this case again, neither of them is able to deliver what they

What they want to deny is the paradoxical nature of modern


democracy and the fundamental tension between the logic of democracy
and the logic of liberalism. They are unable to acknowledge that, while it is indeed the
case that individual rights and democratic self-government are
constitutive of liberal democracy - whose novelty resides precisely in the articulation of those two
traditions - there exists between their respective 'grammars' a tension that
can never be eliminated. To be sure, contrary to what adversaries like Carl Schmitt have argued, this
does not mean that liberal democracy is a doomed regime. Such a tension,
though ineradicable, can be negotiated in different ways. Indeed, a great part
of democratic politics is precisely about the negotiation of that paradox
and the articulation of precarious solutions. 20 What is misguided is the
search for a final rational resolution. Not only can it not succeed, but
moreover it leads to putting undue constraints on the political debate.
Such a search should be recognized for what it really is, another attempt
at insulating politics from the effects of the pluralism of value, this time
by trying to fix once and for all the meaning and hierarchy of the central
liberal-democratic values. Democratic theory should renounce those forms
of escapism and face the challenge that the recognition of the pluralism of
values entails. This does not mean accepting a total pluralism, and some limits need to be put to the kind of
confrontation which is going to be seen as legitimate in the public sphere. But the political nature of
the limits should be acknowledged instead of being presented as
requirements of morality or rationality.
announce.

Consensus Bad Fundamentalism


Democracy is meaningless without dissentabsolutist
consensus shifts antagonisms into other, more brutal spheres
of difference, creating fundamentalism and public apathy
Mouffe 2k Chantal Mouffe, The Democratic Paradox, Verso: New York, 2000, p.
113-115

But this is to miss a crucial point, not only about the primary reality of strife in social life, but also about the

the
specificity of modern democracy lies in the recognition and the
legitimation of conflict and the refusal to suppress it through the
imposition of an authoritarian order. A well-functioning democracy calls
for a confrontation between democratic political positions, and this
requires a real debate about possible alternatives. Consensus is indeed necessary
but it must be accompanied by dissent. There is no contradiction in saying that, as some would
pretend. Consensus is needed on the institutions which are constitutive of democracy. But there will
always be disagreement concerning the way social justice should be
implemented in these institutions. In a pluralist democracy such a
disagreement should be considered as legitimate and indeed welcome. We
can agree on the importance of 'liberty and equality for all', while
disagreeing sharply about their meaning and the way they should be
implemented, with the different configurations of power relations that this
implies. It is precisely this kind of disagreement which provides the stuff of democratic politics and it is what
integrative role that conflict plays in modern democracy. As I have argued through these essays,

the struggle between left and right should be about. This is why, instead of relinquishing them as outdated, we

When political frontiers become blurred, the


dynamics of politics is obstructed and the constitution of distinctive
political identities is hindered. Disaffection towards political parties sets
in and it discourages participation in the political process. Alas, as we have begun
to witness in many countries, the result is not a more mature, reconciled society
without sharp divisions but the growth of other types of collective
identities around religious, nationalist or ethnic forms of identification . In
other words, when democratic confrontation disappears, the political in its
antagonistic dimension manifests itself through other channels.
Antagonisms can take many forms and it is illusory to believe that they
could ever be eliminated. This is why it is preferable to give them a
political outlet within an 'agonistic' pluralistic democratic system. The
deplorable spectacle provided by the USA with the trivialization of political
stakes provides a good example of the degeneration of the democratic
public sphere. Clinton's sexual saga was a direct consequence of this new
kind of bland, homogenized political world resulting from the effects of his
strategy of triangulation. Sure, it allowed him to gain a second term by neutralizing his adversaries
thanks to skilfully drawing on republican ideas that resonated with voters and
articulating them with leftist policies on abortion and education . But at the
cost of further impoverishing an already weak political public sphere . One
should realize that a lack of democratic contestation over real political
alternatives leads to antagonisms manifesting themselves under forms
that undermine the very basis of the democratic public sphere. The
development of a moralistic discourse and the obsessive unveiling of
scandals in all realms of life, as well as the growth of various types of
should redefine those categories.

religious fundamentalism, are too often the consequence of the void


created in political life by the absence of democratic forms of
identification informed by competing political values. Clearly the problem is not
limited to the United States. A look at other countries where, because of different traditions, the sexual card cannot

the crusade against


corruption and shabby deals can play a similar role in replacing the
missing political line of demarcation between adversaries. In other circumstances
yet, the political frontier might be drawn around religious identities or
around non-negotiable moral values, as in the case of abortion, but in all cases what
this reveals is a democratic deficit created by the blurring of the left/right
divide and the trivialization of the political discourse.
be played in the same way as in the Anglo-American world shows that

Consensus Bad Populism


Emptying the political leads to violent populist outbursts like
the Tea Party
Mouffe 2k Chantal Mouffe, The Democratic Paradox, Verso: New York, 2000, p.

116
Another, even more worrying consequence of the democratic deficit linked to the
obsession with centrist politics is the increasing role played by populist
right-wing parties. Indeed, I submit that the rise of this type of party should be
understood in the context of the 'consensus at the centre' form of politics
which allows populist parties challenging the dominant consensus to
appear as the only anti-Establishment forces representing the will of the
people. Thanks to a clever populist rhetoric, they are able to articulate
many demands of the popular sectors scorned as retrograde by the
modernizing elites and to present themselves as the only guarantors of
the sovereignty of the people. Such a situation , I believe, would not have been
possible had more real political choices been available within the
traditional democratic spectrum.

Third Way BadInequality


Third-way reformism naturalizes social hierarchies and
decimates radical political will
Mouffe 2k Chantal Mouffe, The Democratic Paradox, Verso: New York, 2000, p.
14-15

the 'third way' discussed in Chapter


is a 'politics without adversary' which pretends that all
interests can be reconciled and that everybody - provided , of course, that they
identify with 'the project' - can be part of 'the people'. In order to justify
acceptance of the current neo-liberal hegemonywhile pretending to
remain radical - the 'third way' mobilizes a view of politics which has
evacuated the dimension of antagonism and postulates the existence of a
'general interest of the people' whose implementation overcomes the
winners/losers form of resolution of conflicts. The sociological background of such a thesis
On the political level a similar phenomenon is to be found in the case of
5. I argue that it

is that the cycle of confrontational politics that has been dominant in the West since the French Revolution has
come to an end. The left/right distinction is now irrelevant, since it was anchored in a social bipolarity that has
ceased to exist. For theorists like Anthony Giddens, the left/right dividewhich he identifies with old-style social
democracy versus market fundamentalism - is an inheritance of 'simple modernization' and has to be transcended.
In a globalized world marked by the development of a new individualism, democracy must become 'dialogic'. What
we need is a 'life politics' able to reach the various areas of personal life, creating a 'democracy of the emotions'.

What is missing in such a perspective is any grasp of the power relations


which structure contemporary post-industrial societies. There is no
denying that capitalism has been radically transformed, but this does not
mean that its effects have become more benign; far from it. We might have given up the
idea of a radical alternative to the capitalist system, but even a renewed and modernized
social democracy - which the third way claims to be - will need to challenge the
entrenched wealth and power of the new class of managers if it wants to
bring about a fairer and more accountable society. The kind of social
unanimity which is the trademark of Blairism is only conducive to the
maintenance of existing hierarchies. No amount of dialogue or moral
preaching will ever convince the ruling class to give up its power. The
state cannot limit itself to dealing with the social consequences of market
failures.

Third Way BadPower


You cant wish away incommensurabilitythird way politics
accedes to hierarchies of power, making them invisible and
more rigid
Mouffe 2k Chantal Mouffe, The Democratic Paradox, Verso: New York, 2000, p.
110-112
What is really the problem with the advocates of the 'radical centre' is , I
believe, their claim that a left/right divide , a heritage of 'simple modernization', is not
relevant any more in times of 'reflexive modernization'. By asserting that
a radical politics today should transcend this divide and conceive democratic
life as a dialogue, they imply that we now live in a society which is no
longer structured by social division. Nowadays politics operates supposedly on
a neutral terrain and solutions are available that could satisfy everybody.
Relations of power and their constitutive role in society are obliterated
and the conflicts that they entail reduced to a simple competition of
interests that can be harmonized through dialogue. This is the typical
liberal perspective that envisages democracy as a competition among
elites, making adversary forces invisible and reducing politics to an
exchange of arguments and the negotiation of compromises. I submit that to
present such a view of politics as 'radical' is really disingenuous , and that
instead of being conducive to more democracy the radical centrism advocated
by New Labour is in fact a renunciation of the basic tenets of radical politics. The
central flaw of the attempt to modernize social democracy by third way theorists is that it is based on the
illusion that, by not defining an adversary, one can side-step fundamental
conflicts of interests. Social democrats never made that mistake. As Mike Rustin points out, social
democracy, in both its right- and left-wing variants, always had capitalism as one of its antagonists. and its task was

The third
way approach, on the contrary, is unable to grasp the systemic connections
existing between global market forces and the variety of problems - from
exclusion to environmental risks - that it pretends to tackle. Indeed, the main
to confront holistically the systemic problems of inequality and instability generated by capitalism.2

shortcoming of Giddens's analysis is that he appears to be unaware of the drastic measures that would be required
to put most of his proposals into practice.

It is all very nice to announce that there


should be 'no rights without responsibilities' or 'no authority without
democracy', but how is one going to put such programmes into practice
without profoundly challenging the existing structures of power and
authority? Without calling for the sort of total overthrow of capitalism
advocated by some Marxists, one can surely acknowledge that some form
of anti-capitalist struggle cannot be eliminated from a radical politics
aiming at the democratization of society, and that without the
transformation of the prevalent hegemonic configuration little change will
be possible.

Topicality Bad
Rational linguistic agreement is impossiblejudging
interpretations as most reasonable is reflective of particular
forms of life which excludes difference. Democracy entails
reaching out to the other, not pushing them away.
Mouffe 2k Chantal Mouffe, The Democratic Paradox, Verso: New York, 2000, p.

64-66
According to the contextualist approach, Iiberal-democratic institutions
must be seen as defining one possible political 'language-game' -among
others. Since they do not provide the rational solution to the problem of
human coexistence, it is futile to search for arguments in their favour
which would not be 'context-dependent' in order to secure them against
other political language-games. By envisaging the issue according to a Wingensteinian
perspective, such an approach brings to the fore the inadequacy of all
attempts to give a rational foundation to Iiberal-democratic principles by
arguing that they would be chosen by rational individuals in idealized
conditions like the 'veil of ignorance' (Rawls) or the 'ideal speech situation' (Haber- mas). As Peter Winch has
indicated with respect to Rawls, 'The "veil of ignorance" that characterizes his position runs foul of Wittgenstein's

what is "reasonable" cannot be characterized independently of the


content of certain pivotal "judgments" '7 For his pan, Richard Rorty - who proposes a 'neopragmatic' reading of Wittgenstein - has affirmed, taking issue with Apel and Habermas, that it is not
possible to derive a universalistic moral philosophy from the philosophy of
language. There is nothing, for him, in the nature of language that could
serve as a basis for justifying to all possible audiences the superiority of
liberal democracy. He declares: 'We should have to abandon the hopeless task
of finding politically neutral premises, premises which can be justified to
anybody, from which to infer an obligation to pursue democratic politics .'8
point that

He considers that envisaging democratic advances as if they were linked to progress in rationality is not helpful,
and that we should stop presenting the institutions of liberal western societies as the solution that other people will
necessarily adopt when they cease to be 'irrational' and become 'modern'. Following Wittgenstein, he sees the

To call somebody irrational in this


'is not to say that she is not making proper use of her mental
faculties. It is only to say that she does not seem to share enough beliefs
and desires with one to make conversation with her on the disputed point
fruitful.'9 Approaching democratic action from a Wittgensteinian point of view can therefore help us to pose the
question of allegiance to democracy in a different way. Indeed, we are led to acknowledge that democracy
does not require a theory of truth and notions like unconditionality and
universal validity but a manifold of practices and pragmatic moves aiming
at persuading people to broaden the range of their commitments to
others, to build a more inclusive community. Such a shift in perspective reveals that, by
question at stake as one not of rationality but of shared beliefs.
context, he states,

putting an exclusive emphasis on the arguments needed to secure the Iegitimacy of liberal institutions, recent

The real issue is not to find


arguments to justify the rationality or universality of liberal democracy
that would be acceptable by every rational or reasonable person. Liberal
democratic principles can only be defended as being constitutive of our
form of life, and we should not try to ground our commitment to them on
something supposedly safer. As Richard Flathman - another political theorist influenced by
Wittgenstein - indicates, the agreements that exist on many features of liberal
democracy do not need to be supported by certainty in any of the
moral and political theorists have been asking the wrong question.

philosophical senses. In his view, 'Our agreements in these judgements constitute the language
of our politics. It is a language arrived at and continuously modified through no
less than a history of discourse, a history in which we have thought about,
as we became able to think in, that language."10

Competing linguistic interpretations is insufficient for


constructing inclusive debatespolitics must be opened up to
rearticulation on behalf of other forms of lifedemocratic
ethos is a prior question to the application of a rule
Mouffe 2k Chantal Mouffe, The Democratic Paradox, Verso: New York, 2000, p.
67-69
to have agreements in
opinions, there must first be agreement on the language used. And the fact that
he stresses that those agreements in opinions are agreements in forms of life. As
he says: So you are saying that human agreement decides what is true and what is false. It is what human
beings say that is true and false; and they agree in the language they use.
That is not agreement in opinions but in forms of life. 12 With respect to the question
of 'procedures', which is the one that I want to highlight here, this points to the necessity for a
considerable number of 'agreements in judgements' to already exist in a
society before a given set of procedures can work. Indeed, according to Wittgenstein,
to agree on the definition of a term is not enough , and we need agreement
in the way we use it. He puts it in the following way: 'if language is to be a means of
communication there must be agreement not only in definitions but also
(queer as this may sound) in judgements'.12 This reveals that procedures only exist as
complex ensembles of practices. Those practices constitute specific forms
of individuality and identity that make possible the allegiance to the
procedures. It is because they are inscribed in shared forms of life and
agreements in judgements that procedures can be accepted and followed.
The crucial idea provided by Wittgenstein in this field is when he asserts that

They cannot be seen as rules that are created on the basis of principles and then applied to specific cases.

Rules, for Wittgenstein, are always abridgements of practices, they are


inseparable from specific forms of life. The distinction between procedural
and substantial cannot therefore be as clear as most liberal theorists would
have it. In the case of justice, for instance, it means that one cannot oppose, as so many liberals do,
procedural and substantial justice without recognizing that procedural
justice already presupposes acceptance of certain values. It is the liberal
conception of justice which posits the priority of the right over the good, but
this is already the expression of a specific good. Democracy is not only a
matter of establishing the right procedures independently of the practices
that make possible democratic forms of individual- ity. The question of the
conditions of existence of democratic forms of individuality and of the
practices and language-games in which they are constituted is a central
one, even in a liberal-democratic society where procedures play a central role. Procedures always
involve substantial ethical commitments. For that reason they cannot work
properly if they are not supported by a specific form of ethos. This last point is
very important, since it forces us to acknowledge something that the dominant liberal model is unable to recognize,

a liberal-democratic conception of justice and liberal-democratic


institutions require a democratic ethos in order to function properly and
maintain themselves. This is, for instance, precisely what Habermas's discourse theory of procedural
namely, that

democracy is unable to grasp because of the sharp distinction that Habermas wants to draw between moralpractical discourses and ethical-practical discourses. It is not enough to state as he now does, criticizing Apel, that a

discourse theory of democracy cannot be based only on the formal pragmatic conditions of communication and that
it must take account of legal, moral, ethical and pragmatic argumentation.

The objectivity of cold rule-application produces violent


exclusion of other forms-of-lifeagonistic democracy consists
in fostering different interpretations of the rules
Mouffe 2k Chantal Mouffe, The Democratic Paradox, Verso: New York, 2000, p.
69-74

My argument is that, by providing a practice-based account of Rationality, Wittgenstein in his later work opens a
much more promising way for thinking about political questions and for envisaging the task of a democratic politics

In the present conjuncture, characterized by


an increasing disaffection towards democracy - despite its apparent triumph - it is
vital to understand how a strong adhesion to democratic values and
institutions can be established and why rationalism constitutes an
obstacle to such an understanding. It is necessary to realize that it is not by offering
sophisticated rational arguments and by making context-transcendent
truth claims about the superiority of liberal democracy that democratic
values can be fostered. The creation of democratic forms of individuality is
a question of identification with democratic values, and this is a complex
process that takes place through a manifold of practices, discourses and
language-games. A Wittgensteinian approach in political theory could play an important role in the
than the rationalist-universalist framework.

fostering of democratic values because it allows us to grasp the conditions of emergence of a democratic

Giving grounds, however, justifying the evidence, comes


to an end - but the end is not certain propositions striking us immediately
as true, i.e. it is not a kind of seeing on our part; it is our acting, which lies at the bottom
of the language-game.13 For him, agreement is established not on
significations (Meinungen) but on forms of life (Lebensform). It is Einstimmung, fusion of
voices made possible by a common form of life, not Einverstand, product of
reason - like in Habermas. This, I believe, is of crucial importance and it not only indicates the nature of every
consensus but also reveals its limits: Where two principles really do meet which cannot be
reconciled with one another, then each man declares the other a fool and an
heretic. I said I would 'combat' the other man. - but wouldn't I give him
reasons? Certainly; but how far do they go? At the end of reasons comes
persuasion.14 I take this emphasis on the limits of giving reasons to constitute an important starting point for
consensus. As Wittgenstein says:

elaborating an alternative to the current model of 'deliberative democracy' with its rationalistic conception of
communication and its misguided search for a consensus that would be fully inclusive. Indeed, I see the 'agonistic
pluralism' that I have been advocating15 as inspired by a Wittgensteinian mode of theorizing and as attempting to
develop what I take to be one of his fundamental insights: grasping what it means to follow a rule. At this point in
my argumentation, it is useful to bring in the reading of Wittgenstein proposed by James Tully because it chimes
with my approach. Tully is interested in showing how Wittgenstein's philosophy represents an alternative worldview
to the one that informs modern constitutionalism, so his concerns are not exactly the same as mine. But there are
several points of contact and many of his reflections are directly relevant for my purpose. Of particular importance
is the way he examines how in the Philosophical lnvestigations, Wittgenstein envisages the correct way to
understand general terms. According to Tully, we can find two lines of argument. The first consists in showing that

'understanding a general term is not a theoretical activity of interpreting


and applying a general theory or rule in particular cases' .16 Wittgenstein indicates,
using examples of signposts and maps, how I can always be in doubt about the way I
should interpret the rule and follow it. He says, for instance: A rule stands there
like a sign-post. - Does it show which direction I am to take when I have
passed it; whether along the road or the footpath or cross-country? But
where is it said which way I am to follow it; whether in the direction of its
finger or (e.g.) in the opposite one?17 As a consequence, notes Tully, a general rule
cannot 'account for precisely the phenomenon we associate with
understanding the meaning of a general term: the ability to use a general

term, as well as to question its accepted use, in various circumstances


without recursive doubts' .18 This should lead us to abandon the idea that
the rule and its interpretation 'determine meaning' and to recognize that
understanding a general term does not consist in grasping a theory but
coincides with the ability of using it in different circumstances. For Wittgenstein,
'obeying a rule' is a practice and our understanding of rules consists in the
mastery of a technique. The use of general terms is therefore to be seen as intersubjective 'practices'
or 'customs' not that different from games like chess or tennis. This is why Wittgenstein insists that it is a
mistake to envisage every action according to a rule as an 'interpretation'
and that 'there is a way of grasping a rule which is not an interpretation, but
which is exhibited in what we call "obeying the rule" and "going against it"
in actual cases'. 19 Tully considers that the wide-ranging consequences of this point are missed when one
affirms, like Peter Winch, that people using general terms in daily activities are still following rules but that those
rules are implicit or background understandings shared by all members of a culture. He argues that this is to retain
the view of communities as homogeneous wholes and to neglect Wittgenstein's second argument, which consists in

'the multiplicity of uses is too various, tangled, contested and


creative to be governed by rules'.20 For Wittgenstein, instead of trying to reduce
all games to what they must have in common, we should 'look and see
whether there is something that is common to all' and what we will see is
'similarities, relationships, and a whole series of them' whose result
constitutes 'a compli- cated network of similarities overlapping and crisscrossing', similarities that he characterizes as 'family resemblances' .21 I
submit that this is a crucial insight which undermines the very objective that those who advocate the
'deliberative' approach present as the aim of democracy: the establishment of a rational consensus on
universal principles. They believe that through rational deliberation an impartial
standpoint could be reached where decisions would be taken that are
equally in the interests of all.22 Wittgenstein, on the contrary, suggests another view. If we follow
his lead, we should acknowledge and valorize the diversity of ways in which
showing that

the 'democratic game' can be played, instead of trying to reduce this


diversity to a uniform model of citizenship. This would mean fostering a
plurality of forms of being a democratic citizen and creating the
institutions that would make it possible to follow the democratic rules in a
plurality of ways. What Wittgenstein teaches us is that there cannot be one single best,
more 'rational' way to obey those rules and that it is precisely such a
recognition that is constitutive of a pluralist democracy. 'Following a rule' ,
says Wittgenstein, 'is analogous to obeying an order. We are trained to do so; we
react to an order in a particular way. But what if one person reacts in one
way and another in another to the order and the training? Which one is
right?'23 This is indeed a crucial question for democratic theory. And it cannot be resolved, pace the
rationalists, by claiming that there is a correct understanding of the rule that
every rational person should accept. To be sure, we need to be able to distinguish between 'obeying the rule' and

space needs to be provided for the many different practices


in which obedience to the democratic rules can be inscribed . And this should
not be envisaged as a temporary accommodation, as a stage in the process leading to the realization of the
rational consensus, but as a constitutive feature of a democratic society.
Democratic citizenship can take many diverse forms and such a diversity,
far from being a danger for democracy, is in fact its very condition of
existence. This will, of course, create conflict and it would be a mistake to
expect all those different understandings to coexist without dashing. But
this struggle will not be one between 'enemies' but among 'adversaries',
since all participants will recognize the positions of the others in the
'going against it'. But

contest as legitimate ones. Such an understanding of democratic politics, which is precisely what I
call 'agonistic pluralism', is unthinkable within a rationalistic problematic
which, by necessity, tends to erase diversity. A perspective inspired by Wittgenstein, on the
contrary, can contribute to its formulation, and this is why his contribution to democratic thinking is invaluable.

AT: Rules k2 Democracy


Shared principles, like the value of debate as an activity, are
key to agonistic discussion, but the terms and meanings of
those principles must remain contestedwhen the rules
become frozen and timeless, debate loses its political
character
Mouffe 2k Chantal Mouffe, professor in the Department of Politics and
International Relations at the University of Westminster, Deliberative Democracy or
Agonistic Pluralism, Reihe Politikwissenschaft Political Science Series 72, December
2000, probably found on google scholar
Once the theoretical terrain has been delineated in such a way, we can begin formulating an alternative to both the
aggregative and the deliberative model, one that I propose to call agonistic pluralism.32 A first distinction is
needed in order to clarify the new perspective that I am putting forward, the distinction between politics and the

By the political, I refer to the dimension of antagonism that is


inherent in human relations, antagonism that can take many forms and
emerge in different type of social relations. Politics, on the other hand,
indicates the ensemble of practices, discourses and institutions which
seek to establish a certain order and organize human coexistence in
conditions that are always potentially conflictual because they are
affected by the dimension of the political. I consider that it is only when we
acknowledge the dimension of the political and understand that
politics consists in domesticating hostility and in trying to defuse the
potential antagonism that exists in human relations, that we can pose what I
take to be the central question for democratic politics. This question , pace the
rationalists, is not how to arrive at a consensus without exclusion, since this
would imply the eradication of the political. Politics aims at the creation of
unity in a context of conflict and diversity; it is always concerned with the
creation of an us by the determination of a them. The novelty of
democratic politics is not the overcoming of this us/them opposition
which is an impossibility but the different way in which it is established.
The crucial issue is to establish this us/them discrimination in a way that is
compatible with pluralist democracy. Envisaged from the point of view of
agonistic pluralism, the aim of democratic politics is to construct the
them in such a way that it is no longer perceived as an enemy to be
destroyed, but an adversary, i.e. somebody whose ideas we combat but
whose right to defend those ideas we do not put into question. This is the real
political.

meaning of liberal democratic tolerance, which does not entail condoning ideas that we oppose or being indifferent

This
category of the adversary does not eliminate antagonism , though, and it
should be distinguished from the liberal notion of the competitor with which it is
sometimes identified. An adversary is an enemy, but a legitimate enemy, one with
whom we have some common ground because we have a shared adhesion
to the ethico-political principles of liberal democracy : liberty and equality. But we
disagree on the meaning and implementation of those principles and
such a disagreement is not one that could be resolved through
deliberation and rational discussion. Indeed, given the ineradicable pluralism of value, there is
to standpoints that we disagree with, but treating those who defend them as legitimate opponents.

not rational resolution of the conflict, hence its antagonistic dimension.33 This does not mean of course that
adversaries can never cease to disagree but that does not prove that antagonism has been eradicated. To accept
the view of the adversary is to undergo a radical change in political identity. It is more a sort of conversion than a

process of rational persuasion (in the same way as Thomas Kuhn has argued that adherence to a new scientific

Compromises are, of course, also possible; they are part and


parcel of politics; but they should be seen as temporary respites in an
ongoing confrontation. Introducing the category of the adversary requires complexifying the notion of
paradigm is a conversion).

antagonism and distinguishing it from agonism. Antagonism is struggle between enemies, while agonism is struggle
between adversaries. We can therefore reformulate our problem by saying that envisaged from the perspective of

the aim of democratic politics is to transform antagonism


into agonism. This requires providing channels through which collective passions will be given ways to
agonistic pluralism

express themselves over issues, which, while allowing enough possibility for identification, will not construct the
opponent as an enemy but as an adversary. An important difference with the model of deliberative democracy, is
that for agonistic pluralism, the prime task of democratic politics is not to eliminate passions from the sphere of
the public, in order to render a rational consensus possible, but to mobilize those passions towards democratic
designs.

AT: Coalitions/Krishna
Political viability is a secondary issue behind the project of
identifying a political adversaryotherwise too much ground is
ceded to conservative elites, making social change ineffectual
Mouffe 2k Chantal Mouffe, The Democratic Paradox, Verso: New York, 2000, p.
121-122
Radical politics cannot be located at the centre because to be radical - as
Margaret Thatcher, unlike Tony Blair, very well knew - is to aim at a profund transformation of
power relations. This cannot be done without drawing political frontiers
and defining an adversary or even an enemy. Of course a radical project cannot be successful
without winning over a wide variety of sectors. All significant victories of the left have always been the result of an
alliance with important sectors of the middle classes whose interests have been articulated to those of the popular

an alliance is vital for the formulation of a radical


project. But this does not mean that such an alliance requires taking the
middle ground and trying to establish a compromise between neoliberalism and the groups that it oppresses. There are many issues
concerning the provision of decent public services and the creation of
good conditions of life on which a broad alliance could be established.
However, this cannot take place without the elaboration of a new hegemonic
project that would put again on the agenda the struggle for equality which
has been discarded by the advocates of neo-liberalism. Perhaps the clearest sign of
New Labour's renunciation of its left identity is that it has abandoned such a struggle for equality. Under the
pretence of formulating a modern, post-social- democratic conception of
equality, Blairites have eschewed the language of redistribution in order
to speak exclusively in terms of inclusion and exclusion. In their view, the majority of
sector. Today more than ever such

people belong to the middle classes: the only exceptions are a small elite of very rich on one side, and those who

This new social structure is what provides the basis for


the 'consensus at the centre' that they are advocating. Here again we can see that
their main tenet is that society is no longer structured through unequal
power relations. By redefining the structural inequalities systematically
produced by the market system in terms of 'exclusion' they eschew any
type of structural analysis of their causes and side-step the fundamental
question of what needs to be done to tackle them. As if the very condition
for inclusion of the excluded did not require at the very least a new mode
of regulation of capitalism which will permit a drastic redistribution and a
correction of the profound inequalities which the neo-liberal long decade
has brought about.
are 'excluded' on the other.

AT: Bahktin/Dialogical Exchange


Dialogical exchange ignores the antagonistic nature of social
relations which makes progressive exchange possiblethis
results in a utopian project of communication that stifles
difference
Mouffe 2k Chantal Mouffe, The Democratic Paradox, Verso: New York, 2000, p.

129-131
The critique of the consensus approach elaborated in this collection of essays should not be
understood as an endorsement of the view widespread among some 'postmodern' thinkers that democratic politics

should be envisaged as an 'endless conversation' in which one should


constantly try to enter into dialogical relations with the 'Other'. To be sure, those
who advocate such a view usually insist, as I do, on the need to acknowledge 'differences' and on the impossibility

in the end, like the deliberative


model, they are unable to come to terms with 'the political' in its
antagonistic dimension. This is not to underestimate their important divergences. While the
of complete reabsorption of alterity. However, I think that

deliberative democrats, with their emphasis on impartiality and rational consensus, tend to formulate the ends of
democratic politics in the vocabulary of Kantian moral reasoning, the second view eschews the language of
universal morality and envisages democracy not as a deontological but as an 'ethical' enterprise, as the unending
pursuit of the recognition of the Other. To put it a bit schematically, we could speak of the opposition between
moral-universalistic and ethical-particularistic approaches. The vocabulary of those who defend the 'ethical'
perspective comes from a diversity of philosophical sources: Levinas, Arendt, Heidegger or even Nietzsche, and

what is missing in all of them - as in the


deliberative approach - is a proper reflection on the moment of 'decision'
which characterizes the field of politics. This has serious consequences, since it is
precisely those decisions - which are always taken in an undecidable
terrain - which structure hegemonic relations. They entail an element of
force and violence that can never be eliminated and cannot be adequately
apprehended through the sole language of ethics or morality. We need a
reflection of the political proper. Let's be dear. I am not arguing that politics should be dissociated
there are significant differences among them; but

from ethical or moral concerns, but that their relation needs to be posed in a different way. I would like to suggest
that this cannot be done without problematizing the nature of human sociability which informs most modern
democratic political thinking. To grasp the shortcomings of the dominant view we need to go back to its origins: the
period of the Enlightenment. A useful guide for such an enquiry is provided by Pierre Saint-Amand in The Laws of
Hostility, a book where he proposes a political anthropology of the Enlightenment. 1 By scrutinizing the writings of
Montesquieu, Voltaire, Rousseau, Diderot and Sade through the perspective developed by Rene Girard, he brings to
the fore the key role played by the logic of imitation in their conception of sociability while, at the same time,

in their attempt to ground politics on


Reason and Nature, the Philosophes of the Enlightenment were led to
present an optimistic view of human sociability, seeing violence as an
archaic phenomenon that does not really belong to human nature.
According to them, antagonistic and violent forms of behaviour , everything that
is the manifestation of hostility, could be eradicated thanks to the progress of
exchange and the development of sociability. Theirs is an idealized view of
sociability that only acknowledges one side of what constitutes the
dynamics of imitation. Pierre Saint Amand indicates how in the Encyclopedia human reciprocity is
unveiling its repressed dimension. He shows how,

envisaged as aiming exclusively at the realization of the good. This is possible because only one part of the mimetic

if one recognizes the


ambivalent nature of the concept of imitation, its antagonistic dimension
can be brought to light and we get a different picture of sociability . The
importance of Girard is that he reveals the conflictual nature of mimesis, the
double bind by which the same movement that brings human beings
together in their common desire for the same objects is also at the origin
affects, those linked to empathy, are taken into account. However,

of their antagonism. Rivalry and violence, far from being the exterior of
exchange, are therefore its ever-present possibility. Reciprocity and
hostility cannot be dissociated and we have to realize that the social order
will always be threatened by violence. By refusing to acknowledge the
antagonistic dimension of imitation, the Philosophes failed to grasp the
complex nature of human reciprocity. They denied the negative side of
exchange, its dissociating impulse. This denial was the very condition for
the fiction of a social contract from which violence and hostility would
have been eliminated and where reciprocity could take the form of a
transparent communication among participants. Although in their writings many
of them could not completely elude the negative possibilities of imitation,
they were unable to formulate conceptually its ambivalent character. It is
the very nature of their humanistic project - the ambition to ground the
autonomy of the social and to secure equality among human beings - that
led them to defend an idealized view of human sociability.

Violence UQ

AT: PinkerGeneral
Pinker never specifies what violence is
Laws 12 Ben Laws, Against Pinker's Violence, CTheory.net, 3/21/2012,
http://www.ctheory.net/articles.aspx?id=702 typo corrected in brackets
What is striking about a book dedicated to explaining the developments of
violence across time is that [it] does not begin with any kind of formal
definition -- there is a clear absence in fact. It is implicitly assumed that
we the reader know exactly what constitutes a violent act. Pinker references a
number of historical trends, events and literary sources that suggest the past was a remarkably vicious and brutal
time to have lived -- the Greeks, Romans, the Hebrew bible and early Christendom all feature to support this. But

at no point does he extrapolate what kind of violence he is attempting to


locate.

The 20th century was unprecedentedly violent in absolute


terms
Laws 12 Ben Laws, Against Pinker's Violence, CTheory.net, 3/21/2012,

http://www.ctheory.net/articles.aspx?id=702
Pinker subscribes to a certain voice of 'Truth', namely one which flights
the steady decline of violence over time. Yet, if we take a 'perspectivist'
stance in relation to matters of truth would it not be possible to argue the
direct inverse of Pinker's historical narrative of violence? Have we in fact become
even more violent over time? Each interpretation could invest a certain
stake in 'truth' as something fixed and valid -- and yet, each view could be
considered misguided. What would this alternative history look like? It
could be equally as systematic; it could be equally scientific, full of
'reasoned' argument and as enchanted with modernity, as Pinker's thesis. It could
start by stating that in fact the devils of our nature have outmanoeuvred
the angels. As evidenced by the multiple atrocities of the 20th century -the only century where the world's great powers declared war on each
other, twice. It is estimated that over 70 million combatants were armed
and sent to fight in the First World War, and new advancements in
technology allowed massive losses of life on an unprecedented scale . Some
twenty years later, World War II signalled the biggest conflict (in terms of death
toll) to be historically recorded -- and what separates it from other great
historical wars is the sheer concentration of deaths (estimates of 60
million are common) in the space of only 6 years. Mines, bombs, nuclear
warfare, increasingly accurate projectiles, gas and chemicals, jets and
apaches effectively created an expanded spectrum of ways to inflict death
from a greater distance.

AT: PinkerDemocratic Peace


Democracies have been terribly violent
Laws 12 Ben Laws, Against Pinker's Violence, CTheory.net, 3/21/2012,
http://www.ctheory.net/articles.aspx?id=702

Pinker argues that a shift towards democratic rule and increased wealth in the west has directly correlated with the

The formation of democratic nation-states


was one of the principal drivers of violence of the last century, involving
ethnic cleansing in inter-war Europe, post-colonial states and the postcommunist Balkans. Steadily-growing prosperity may act as a kind of
tranquilliser, but there is no reason to think the increase of wealth can go
on indefinitely -- and when it falters violence will surely return. In quite different
ways, attacks on minorities and immigrants by neo-fascists in Europe, the
popular demonstrations against austerity in Greece and the English riots
of the past summer show the disruptive and dangerous impact of sudden
economic slowdown on social peace. [4]
decrease in violence, but John Gray counters that:

AT: PinkerFoucault K
History is a matter of interpretationPinkers reductionist
account of violence conceals instances of violence excessive of
classic explication and erases essential historical differences
Laws 12 Ben Laws, Against Pinker's Violence, CTheory.net, 3/21/2012,
http://www.ctheory.net/articles.aspx?id=702 typo corrected in brackets
What do we achieve by placing our morality and values onto the Romans,
Greeks, Egyptians, Victorians, Byzantines, Mayans etc? Is it attempting to
compare the incomparable? But, is this not, how a misguided history
[begins]? It assumes that 'words have kept their meaning, that desires
still pointed in a single direction, and that ideas retained their logic, [and
it ignores the fact that] the world of speech and desires has known
invasions, struggles, plundering, disguises, ploys. ' [8] Indeed, to comprehend
and interpret the ideas of a period we have to stare into the face of the
singularity of individual events -- without sating that tempting urge for
finality, for grand themes across the evidence. To return to violence, which Pinker does not
openly define, we can intuit (roughly) from his chapters that he means physical force (murders, torture, hand-to-

Pinker's undefined
definition and approach to violence enables his quantification of it -- so
that across centuries and millennia certain forms and intrusions of
violence can be correlated, evaluated and (re)interpreted. It is this
scientific quantification project and statistical reduction that forms the
basis of his thesis. He will use graphs, charts and tables to reiterate a
numbers game -- all echoing decrease. But what would it mean to be a
violent Roman compared to a violent Victorian? And how can we begin to
compare this to a violent modern man? To each historical period there
must be a corresponding understanding and comprehension of exactly
what it meant to be violent. If we look back at history through a modern
lens we are destined to find horrific images at every turn: we see the
alien, the depraved. For each historical age (and the hazy less-discussed boundaries in-between)
we would have to resist the urge to look at violence so directly. By trying
to look for answers in a straight line, we forget to turn. We reveal the all
too 'rational' and 'reasoned' methods of our times. Pinker's science -- and
science as a whole -- is not a value free practise; the way he applies and defends
his position is through the application of a science laden with his ideopolitical position on the spectrum (statistics, logical argumentation, quantification and
reasoning). To understand a specific period we would do better to locate those
sources that surrounded, influenced and were affected by violence:
perhaps we would assemble discordant fragments and a complex
patchwork of effects and sociological trends. We would find that the
nature of violence has not evolved on a stable or constant line. To validate
such transformations our definition(s) of violence must also react and
evolve in an equally intricate way. We might be hard-pressed to find instruments of physical
hand conflicts and assaults, rapes, conquests and wars) doled-out to others. Crucially,

torture in the modern world (speaking in terms of the 'developed' west) and certain kinds of hand-to-hand

it is a giant leap, though not an uncommon one, to draw


overarching conclusions from such an observation. For while certain types
of aggression may have decreased have we not created new forms and
pathways for violence in lieu? These forms may often go beyond the realm
punishment are rarer in our time. Yet

of physicality; we need to be subtle and sensitive to these transitions, for


we can be violent without causing direct physical pain. Pinker Responds Pinker
published a list of responses to criticism targeted at his work; two interesting and relevant questions/answers are
set-out below: Is economic inequality a form of violence? No; the fact that Bill Gates has a bigger house than I do
may be deplorable, but to lump it together with rape and genocide is to confuse moralization with understanding.
Ditto for underpaying workers, undermining cultural traditions, polluting the ecosystem, and other practices that
moralists want to stigmatize by metaphorically extending the term violence to them. It's not that these aren't bad
things, but you can't write a coherent book on the topic of "bad things." [9] What about metaphorical violence, like
verbal aggression? No, physical violence is a big enough topic for one book (as the length of Better Angels makes
clear). Just as a book on cancer needn't have a chapter on metaphorical cancer, a coherent book on violence can't
lump together genocide with catty remarks as if they were a single phenomenon. Pinker executes the same
processes in his response as his does in his book, namely, reasserting a reductive vision of what it means to be

a book could not be written on non-physical


forms of cancer because cancer directly affects the body -- the riposte is
that violence does not have to, and it indeed operates quite indirectly in
this respect. But there is a pitfall in engaging with Pinker's dialogue, as our
(and any other) response will ultimately fall-short, because he has set the
guidelines and has created the validating conditions. We need to reject his
framework entirely.
violent -- an exercise in circularity. Incidentally,

Reject Pinkers linear historyhis absolute understanding of


science is an intersection of power/knowledge which replicates
and exacerbates contemporary violence and ethical disregard
Laws 12 Ben Laws, Against Pinker's Violence, CTheory.net, 3/21/2012,

http://www.ctheory.net/articles.aspx?id=702 typo corrected in brackets


We should create a position against Pinker's narrative -- for his thesis is,
on these terms, violent. He effectively barricades wider forms of
interrogation while also dismissing these voices as 'moralisation' or antiscience. It is a rejection that operates under the strict tenure of his
relation in the power structure -- one that places a paramount on certain
scientific/technical truths. But 'truth is undoubtedly the sort of error that
cannot be refuted because it was hardened into an unalterable form in the
long baking process of history.' [10] A nuanced understanding of violence
should question those so-called direct and logical historical accounts that
lead us wearily to the modern day. Ultimately, Pinker will conclude that: Yet while this planet has
gone cycling on according to the fixed law of gravity, that species has also found ways to bring the numbers down,
and allow a greater and greater proportion of humanity to live in peace and die of natural causes. For all the
tribulations in our lives, for all the troubles that remain in the world, the decline of violence is an accomplishment
we can savour, and an impetus to cherish the forces of civilization and enlightenment that made it possible.[11]

this framing of the


problem by Pinker is predominant and pervasive today. It serves as a justification and it
systematically excludes. It should not be a surprise that this simplified conception of
violence results in a linear thesis. We should see violence as an intense
force that operates in a myriad of sophisticated ways, mediated over
certain times and spaces. A virulent and destructive form of abuse is
neglect, for the perpetrator carries a burden of care for the victim, who is
heavily dependant, but fails to act it out. Neglect can injure a person for
life (emotionally and physically), impairing development with devastating
results, often self-harm and sometimes death. A person suffering from
neglect may often forgo the chance to build personal relationships and
live with independence or choice. By definition, there is no physicality
involved in neglect, but who could deny the violent force of its effects?
Pinker's thesis is an exact sample of neglect, the practise excludes and
Such a violent set of closing thoughts is set-out clearly, but we should observe that

omits, it carries a burden of care, but by staring too directly it fails to see
the vastness and enormity of the problem.

Generic Links

China Link 1NC


Realist depictions of a rising Chinese threat freeze national
identity and geopolitical space to encourage violence while
disrupting trajectories towards cooperation and peace
Lim 12 Kean Fan Lim, What You See Is (Not) What You Get? The Taiwan Question,
Geo-economic Realities, and the China Threat Imaginary, Antipode, Volume 44,
Issue 4, pages 13481373, September 2012, DOI: 10.1111/j.14678330.2011.00943.x

Despite the popularized discourses about globalization and geo-economic integration, states continue to prioritize
national interests through reconfiguring their capacities (see eg Ross 1999; Sassen 2008; Sparke 2006). In this
regard, the prospects of inter-state conflict continue to loom in the background of geo-economic processes aimed at
broadening spaces of/for capital accumulation beyond national confines. Echoing Kant, Waltz (2000:8) argues that
the natural state remains the state of war: under the conditions of international politics, war recurs; the sure way
to abolish war, then, is to abolish international politics. As far as the past decade goes, Waltz's observation
appears prescientwars continue to be waged between and within nation-states, with the primary aim of either

The US responses to the 9/11 terrorist attacks


reaffirms the realist assumption that in the absence of an overarching
cosmopolitan state (or a sovereign state of states), the nation-state is responsible for
security around and within its territorial borders. This entails different
strategies, from building border walls and scaling up border control
overseas (Cowen and Smith 2009) to pre-emptive attacks on and occupation of
other states (Fallows 2006; Gregory 2004; Harvey 2003). National-level provision of
normative security at the global scale is not a natural given, however. This
authority rests on the moral contract of citizenship; individuals, as citizens,
assume the state is supposed to offer protection against foreign threats .
destabilizing or fortifying a state's existential right.
further

As Scott and Carr (1986:83) put it: Let us characterize the responsibility the state owes to its citizenry as the
obligation to manage international uncertainty in the best interest of the citizenry. The obligation, of course, is owed
to the state's citizenry, but it gives purpose and direction to the state's foreign policy. It seems appropriate, then, to
describe the state as the advocate of its citizen's interest in the international world. Inter-state relationships
correspondingly should be regarded as relationships between advocates charged with pursuing the interests of their

A state's threat-evaluation process becomes


problematized when a geo-economic conception of security is adopted,
because a foreign entity that is perceived to be a political or military threat
may simultaneously offer significant economic security to the domestic
citizenry. Indeed, a geo-economic conception of security has probably become a necessity in the context of
contemporary globalization of capital flows. As Strange (1996) correctly observes, myriad economic actors
have been encroaching upon the traditional sovereignty of territorial
states in order to broaden their global influences. Acknowledging the intrinsic
respective clients; their citizenry.

limitations of nation-based geopolitik, Beck (2005) argues that a cosmopolitan politics of golden handcuffsone
that recognizes the importance of international economic interdependency and the corresponding need to
overcome national differencescould in fact better sustain national interests. Within East Asia, geo-economic
strategizing offers a twist to Ross's (1999) geographically deterministic analysis of the regional power of balance
that the physical geographies of East Asia are likely to prevent conflictbecause geo-economics gets at the way in
which a more or less geopolitical phenomenon (of imagining territory as a mode of political intervention and
governance) is closely articulated with a whole series of economic imperatives, ideas and ideologies (Sparke
1998:69). Then again, this does not mean that states are necessarily retreating vis--vis pressures from the global
system of capitalism (paceStrange 1996; cf Sassen 2008). While Harvey and Scott (1989:222, 224) contend that
developments in political economy are fundamentally reducible to a stubborn logic of capitalism shaped by the
real universal qualities of capitalism, the influential role states undertake to contour the form of spaces of/for
capitalist accumulation through geo-economic integration indicates the necessary imbrication of state power in this

Capitalism, in other words, is not a hermetically sealed system; its


geographical expressions are significantly constituted by fusions of statedriven geopolitics and variegated degrees of capitalist realism (cf Brenner 2004;
Peck and Theodore 2007). The picture in East Asia is certainly blurred when capitalist
logic.

systemic pressures overlap state-centric geopolitical concerns . As


aforementioned, while it appears that the US government increasingly needs to
collaborate with its counterpart in Beijing to enhance mutual economic security through
encouraging and managing flows of capital, commodities and people, the discursive constructions
of a China threat have arguably become more amplified in the past few
years than before. Cowen and Smith (2009:25) thus offer a plausible observation that the
geoeconomic conception of security underlines conflicts between the
logics of territorial states and global economic flows, the proliferation of
non-state and private actors entangled in security, and the recasting of
citizenship and social forms. The way US defense policymakers see
China as a potential military threat to Taiwan, which in turn legitimizes
defensive protection through arms sales, is particularly instructive: it might
actually contradict what China construes as peaceful coexistence and
disrupt attempts by actors in China, Taiwan and the US to carve out integrated
economic spaces. Indeed, a major component of US geopolitical strategy in
East Asia is historically entwined with calculations within the Taipei-based
Republic of China (ROC; hereafter Taiwan) government that perceive China as an existential
threat (this historical circumstance is elaborated in the third section). The USROC entwinement is illustrated in
one of the Six Assurances1 the Reagan administration gave to the ROC government in July 1982, which states
that the US would not consult with China in advance before making decisions about U.S. arms sales to Taiwan.
This stance represents a provocation to the Chinese government, as it considers arms sales from foreign parties to
Taiwan an intervention in its domestic affairs. The difference in view between China and the US over Taiwan thus
allows us to develop Cowen and Smith's (2009:34) insight that: Where geopolitics can be understood as a means of
acquiring territory towards a goal of accumulating wealth, geoeconomics reverses the procedure, aiming directly at
the accumulation of wealth through market control. The acquisition or control of territory is not at all irrelevant but
is a tactical option rather than a strategic necessity. To be sure, geopolitical calculation is always available when

Why are these


calculations deemed necessary in spite of the systemic imperatives of
capitalism that shape ChinaUS and China region geo-economic
integration? While I do not claim to offer definitive answers to these questions, I believe that they have
to be critically assessed in order to obtain a more incisive understanding
of East Asian geopolitics and China's changing position in the global
political economy. This will now be addressed in greater detail.
deemed necessary. What, then, figures in the US geopolitical calculations of China today?

China Link
China threat mongering is dangerous, counterfactual, and
hypocriticalhikes tensions and overwhelms potential for
cooperation
Lim 12 Kean Fan Lim, What You See Is (Not) What You Get? The Taiwan Question,
Geo-economic Realities, and the China Threat Imaginary, Antipode, Volume 44,
Issue 4, pages 13481373, September 2012, DOI: 10.1111/j.14678330.2011.00943.x
The identification of national-level threats is never straightforward; it is
often effected on emotional rather than evidential grounds. Within the US,
Japan bashing emerged during the 1980s vis--vis domestic fears of the
waning economic competitiveness of the US and its seeming inability to
confront a flexible post-Fordist future. More recently, the US decision in 2003
to attack Iraq on the premise that it possessed weapons of mass
destruction proved ultimately groundless: no such weapons were found,
while Iraq slipped into anarchy and arguably became a hotbed of terrorist
activities only since (see Fallows 2006; Gregory 2004; Tuathail 2004). Mandel (2008:40) thus rightly
cautions how [t]he political manipulation of enemy images by both government
officials and members of the mass public clouds over the stark realities
surrounding any international enemy predicament. Together, these patterns
create both ambiguity and confusion in dealing with the enemy
component of global threat. In the context of this paper, the critical question is
whether a China threat imaginary is actually produced by forces beyond
China; whether what you see is indeed what you get. Even though the US formally

recognized the People's Republic of China (PRC) as a state in 1979, the recognition was arguably conditioned by
latent suspicions. As Feldman (2007:np) puts it, the Reagan administration continued to put little trust in Chinese
promises to adhere to a peaceful solution regarding Taiwan even as it prepared to sign the 1982 communiqu2
with China. This little trust, Feldman (2007) adds, explains why Reagan gave Taiwan Six Assurances and also
inserted a secret memo in the National Security Files noting that Taiwan's defensive capabilities must be maintained
at a level relative to China's. Reagan's legacy of little trust seems to have permeated subsequent policy
considerations. In 1999, the Pentagon presented several scenarios in its Asia 2025 study that portrayed China as
the most significant threat to American interests in the Asia-Pacific by 2025. A decade on, the US Defense Secretary
Robert Gates offers this analysis of China: In fact, when considering the military-modernization programs of
countries like China, we should be concerned less with their potential ability to challenge the US symmetrically
fighter to fighter or ship to shipand more with their ability to disrupt our freedom of movement and narrow our
strategic options Investments in cyber and anti-satellite warfare, anti-air and anti-ship weaponry, and ballistic
missiles could threaten America's primary way to project power and help allies in the Pacificin particular our

Gates
geographical imagination of China in this speech is predicated on two interrelated assumptions that exemplify a political realist way of seeing. First,
China is not recognized as an ally of the US, although it is clear that the
US is the key driver of such politics of recognition in the first place.
Furthermore, it appears that US military protection is a precondition to qualify as
an ally, a logic which automatically casts states without such
protection as suspect. Second, China's military-modernization process is
ostensibly a threat because such efforts could, in Gates terms, disrupt the
strategic options of the US in East Asia, even when it is entirely
plausible that increased defense spending is to fulfil other valid purposes,
such as replacing obsolete military equipment to address new threats by
terrorists and maritime pirates, and enhancing remuneration packages for
soldiers. Third, America wants to project power on its own terms, which is
forward air bases and carrier strike groups (US Department of Defense 16 September 2009).

why it becomes concerned when so-called non-allies upgrade their


defence technologies. This point is further reaffirmed in the Pentagon's
2010Quadrennial Defense Review: lack of transparency and the nature of China's military development and
decision-making processes raise legitimate questions about its future conduct and intentions within Asia and

the extent to which the questions are


legitimate is clearly a unilateral legal-discursive construction of the US
that reflects the enduring effect of political realism in US security thought.
These assumptions collectively constitute what Bialasiewicz et al (2007; see also Lott 2004)
call America's performative security strategy, through which perceived
insecurities are constructed as ontological facts so that mitigation
measures could be justified. A critical assessment of the motivations behind China's military
beyond (Pentagon 2010:60). However,

modernization policies is thus necessary before it can be ascertained whether a China threat exists. First, while
China is not recognized as a US ally, it does not justify its defense modernization programs through anti-US

For Chinese policymakers, it does appear that the critical issue is


protection and consolidation of its existing territories (more on this in the third
section). According to Luo Yuan, a member of the Chinese People's Political Consultative
Conference (CPPCC) and senior researcher with the Academy of Military
Sciences, China is the only permanent member of the UN Security Council
that has not achieved territorial integrity We need to think more on how
to preserve national integrity. We have no intention of challenging the US
(China Daily 4 March 2010). In terms of defense budget, China's increased 2010 budget, at
around US$78 billion, pales in comparison to the proposed expenditure of the US of US$700 billion.
Furthermore, the request for China to be transparent in its defense policies
is a double-standards practice which undermines the sovereign right of a
country to devise its own policies, since the Pentagon is not the most
transparent or accountable where its own policiesespecially the
supposed right to launch pre-emptive strikesare concerned . If anything,
rhetoric.

then, the massive power gap between China and the US should suffice to allay concerns about the former's so-

the US and
Chinese economies are inextricably intertwined as the Chinese
government currently generates effective demand for US Treasury
financial instruments and holds significant US dollar reserves . In addition,
China's growing geo-economic influence worldwide is contingent on a
strategic investment of its foreign reserves, which means it has every
economic incentive to ensure stability in the global monetary system (see
discussion in Lim 2010). Within the US, however, it is possibly this very geo-economic
integration with China (especially the US Treasury's increasing dependence on Chinese financial capital and
China's importance as an offshore outsourcing destination for US transnational corporations) which triggers
suspicions of China's intentions and which then generate certain reactive
measures to deflect attention from the US economy's deep-seated
problems. In an insightful analysis, Cohen and DeLong (2010:1213) argue that the US has had a wonderful
called threat (cf Al-Rodhan 2007). Second, it is interesting that whilst not allies in name,

opportunity to create new sectors of the future because of the willingness of developing countries like China to
lend it money; what was created, however, was a finance sector that almost bankrupted the economy and
deepened the need for foreign backing to support its quantitative easing monetary solution. Because the need to
borrow more money from abroadand China is so far the biggest creditorcould lead to the end of the global
politico-economic influence of the US, it is perhaps unsurprising that some political actors choose to politicize this
phenomenon. As Waltz (2000:15) puts it, With zero interdependence, neither conflict nor war is possible. With

if China has no plausible


economic motivation to engage in military conflict with the US, the
potential for conflict could be attributed to the unilateral and sustained
willingness of the US to accede to Taiwan's arms purchase requests, in the
integration, international becomes national politics. Then again,

knowledge that China views such arms sales as a clear show of support for what it considers its own province.
Intriguingly, the US framing of its relations with Taiwan could actually be due to an implicit distrust of putative allies
in the East Asian region. Cha (2010:158) theorizes post-World War II US geopolitical alliances with South Korea,

Taiwan and Japan as a form of bilateral powerplay designed to suppress not only the Soviet threat, but also: to
constrain anticommunist allies in the region that might engage in aggressive behavior against adversaries that
could entrap the United States in an unwanted larger war. Underscoring the U.S. desire to avoid such an outcome
was a belief in the domino theorythat the fall of one small country in Asia could trigger a chain of countries falling
to communism. This strategy arguably applies in the present day, despite the demise of the Soviet Union and
China's peaceful integration into the global political economy. For instance, Christensen (1999:50) sees US military
presence in East Asia as resolving a security dilemma triggered by a tendency for one country, affected
profoundly by historically based mistrust, to overreact to another country's acquisition of ostensibly defensive
military equipment. What Christensen (1999) does not emphasize, however, is that the US is also a major exporter
of such equipment, which makes the powerplay logic a doublethink ratiocination. This echoes Cowen and Smith's
(2009:42) aforementioned caveat that geopolitical calculation is always available when deemed necessary. Even
though the Cold War is officially over, Johnson's analysis (2005, in Asia Times Online) strongly suggests that the
powerplay approach remains in full swing: Since the end of the Cold War in 1991, the United States has
repeatedly pressured Japan to revise Article 9 of its constitution (renouncing the use of force except as a matter of
self-defense) and become what US officials call a normal nation America's intention is to turn Japan into what
Washington neo-conservatives like to call the Britain of the Far Eastand then use it as a proxy in checkmating
North Korea and balancing China. Cha's (2010)powerplay thesis thus illustrates how collaborating with the US
through bilateral alliances already implies subjugation to broader US interests, although the US need not
necessarily view subjugation as non-threatening. Asymmetrical bilateral alliances (between a large power and a
smaller ally), as Cha (2010:164) puts it, are actually power instruments of control through which the larger
patron enjoys a great deal of leverage. Such alliances are preferred when larger powers do not want to lose power
vis--vis smaller allies, which highlight the implicit distrust prior to alliance formation. After all, as Bartelson
(1995:164) writes, [s]ecurity

is not primarily an object of foreign policy; before


security can be brought to function as such, it requires a prior
differentiation of what is alien, other or simply outside the state and
therefore threatens it. Building on the insights of Johnson (Asia Times Online 2005) and Cha (2010), I
will explore the implications of this powerplay strategy in the next section by focusing on how the geopolitical
calculations of the USexemplified through the unilaterally crafted TRA and sustained arms sales to Taiwancould
indirectly destabilize the China region and possibly even Sino-US geo-economic formations.

China Framework
Discourse about China shapes perceptions of future actions
and constrains policy responses
Johnston 13 Alastair Iain Johnston, The Governor James Albert Noe and Linda

Noe Laine Professor of China in World Affairs at Harvard, How New and Assertive Is
Chinas New Assertiveness? International Security, Vol. 37, No. 4 (Spring 2013), pp.
748
Why should policymakers and scholars worry about a problematic characterization of Chinese foreign policy?
Putting aside the intellectual importance of accurately measuring the dependent variable in the study of a major

the new assertiveness meme


could contribute to an emerging security dilemma in the U.S.-China
relationship. Talk is consequential for both interstate and intrastate
politics during intensifying security dilemmas and strategic rivalries. How
adversaries are described reverberates in the domestic politics of both
sides.3 The effect is often the narrowing of public discourse . As public discourse
narrows and as conventional wisdoms become habituated, it becomes more
difficult for other voices to challenge policy orthodoxies .4 Similar to the
containment meme in China,5 the new assertiveness meme or others similar to it in the United States
could, in the future, reduce the range of interpretations of Chinese foreign
policy, potentially narrowing policy options available to decisionmakers
powers foreign policy, there are two good reasons. First, if it persists,

(assuming this discourse becomes accepted by national security decisionmakers).

Global Warming Link


Climate science presupposes a metaphysical divide between
humanity and nature that distorts its epistemic validity and
political efficacy
Head and Gibson 12 Lesley Head, University of Wollongong, Australia, and
Chris Gibson, University of Wollongong, Australia, Becoming differently modern:
Geographic contributions to a generative climate politics, Progress in Human
Geography, December 2012 vol. 36 no. 6 699-714
There are a number of interconnected implications here for how we might
think differently about climate change. First, emphasis on the moment of
collision between two separate entities (the impact of humans on climate) has
favoured historical explanations that depend on correlation in time and
space, to the detriment of the search for mechanisms of connection rather
than simple correlation (Head, 2008). This is particularly important to how we think about the future,
since removal of the human is presumably not our solution of first resort. As Hulme (2010a: 270) argues, it is
as irrelevant as it is impossible to find the invisible fault line between
natural and artificial climate. Second, putting the significant explanatory
divide between humans and nature requires the conflation of bundles of
variable processes under the headings human, climate and nature . For
example climatic processes can include everything from astronomical
forcing at 100,000-year timescales to ENSO cycles of a decade or so, and
trends that can be warming, cooling, wetting or drying. In practical terms, taking
apart the climate monolith allows us to consider how mooted
anthropogenic changes leading to future scenarios will take expression in
and through existing patterns of weather and climatic variability (Hulme, 2008).
Taking apart the human monolith forces us to consider exactly what the constituent practices of solutions might be.
For the most part the deconstructive effort is yet to pervade physical geography and archaeology, where human
impacts a conceptualisation that positions humans as outside the system under analysis, as outside nature
remains the dominant, if implicit, conceptualisation of the human-nature engagement over timescales of hundreds
and thousands of years (Head, 2008). Nevertheless, this long-term perspective has provided a crucial underpinning

a key contradiction
persists: we maintain dualistic ways of talking about things (human
impacts, human interaction with environment, anthropogenic climate
change, cultural landscapes, social-ecological systems), while the empirical
evidence increasingly demonstrates how inextricably humans have become
embedded in earth surface and atmospheric processes .
to the identification of anthropogenic climate change in the palaeoclimatic record. So,

Even if science could be 100% objective, translation into policy


contexts carries socio-political challenges causing distortion
and mis-translation
Brace and Geoghegan 11 Catherine Brace, University of Exeter, UK, and

Hilary Geoghegan, University of Exeter, UK, Human geographies of climate change:


Landscape, temporality, and lay knowledges, Progress in Human Geography, vol.
35, no. 3, pp. 284-302, June 2011, doi: 10.1177/0309132510376259
At the heart of the ambiguities surrounding climate change are the differing
definitions of the term used by the United Nations Framework Convention on Climate Change
(UNFCCC) and the Intergovernmental Panel on Climate Change ( IPCC), respectively. The former
defines climate change as a change of climate that is attributed directly or

indirectly to human activity, that alters the composition of the global


atmosphere, and that is in addition to natural climate variability over comparable time
periods (UNFCCC, 2010). In contrast, the IPCC definition of climate change does not
distinguish between natural variability and anthropogenic forcing . The real
material consequences attributed to this lack of clarity are physical, social
and political, including sea-level rise and loss of habitat as well as a
stalemate in international dialogue on climate policy, lack of effective
energy policy, loss of livelihoods and impacts on human health (see, for example,
McMichael et al., 2003; Pielke, 2004; Piguet, 2008). Definitional problems with the key phrase climate
change are compounded by unclear definitions of associated terms such as
climate variation, climate fluctuation and climatic variability which , as Lars
(1993) argues, might not be a problem for a scientific community but creates
difficulties when transferred into a political or socio-economic context to
construct among other things emissions targets, policy instruments, legal frameworks, treaties,
regulations, taxes and subsidies (Leiserowitz, 2005; Osofsky, 2005). In light of this
definitional ambiguity, climate change is, then, simultaneously a reality, an
agenda, a problem and a context. It remains the source of much difficulty
in the science, politics and cultures of climate change . For this reason in this paper we
attempt a more open and inclusive formulation climate and the ways it might change that allows different ways
of knowing to play a legitimate part in framing a relationship with landscape. This formulation draws on the
epistemologies of the social sciences and humanities, and it is this approach that we explain in the next section.

Terminally implicating climate change in terms of thresholdcrossings is a counterproductive totalization which conceals
ongoing violence and constrains policy and local responses
Cupples 12 Julie Cupples, Wild Globalization: The Biopolitics of Climate Change
and Global Capitalism on Nicaragua's Mosquito Coast, Antipode, Volume 44, Issue
1, pages 1030, January 2012, DOI: 10.1111/j.1467-8330.2010.00834.x
In the first world, in both everyday and scientific discourse, climate change is
frequently posited as a transcendent and teleological megahazard , caused by

the prime movers greenhouse gases, which have the potential to wreak havoc and undermine our way of life. While

it is often
described in first world contexts as a future-oriented problem, as one
which will affect future generations if we fail to act . For example, Giddens (2009:1)
both scientists and ordinary people also talk about climate change in a range of tenses,

starts his recent book on climate change by describing it as something which has potentially devastating
consequences for the future. A recent Bolivian blog states that Oxfam America made a serious mistake in its
recent report on indigenous peoples and climate change by tensing its warnings in the future tense (duderino

It is apparent that the biophysical realities which we socially


construct as climate changerising sea levels, drought and flooding,
intensified hurricanes, disappearing iceare of course affecting millions of
humans and nonhumans right now, in Bangladesh, the Sunderbans
Islands, Tuvalu, Papua New Guinea and Central America. The repeated
positing of climate change as a future-oriented problem constitutes an
insidious erasure of those killed and displaced by climate-related disasters
at the present time. Dominant approaches to climate change are clearly a
key part of the neocolonial global order, in which the deaths of third world
inhabitants in disasters are more acceptable, more justifiable, than the
future potential deaths of first world people who haven't been born yet .4
However, such biopower, like global war, as discussed by Hardt and Negri (2004:20), must not only
bring death but must also produce and regulate life. Flusty (2004:7), who works his
2009).

argument about globalization around the concept of de-coca-colonization, urges us to shift: our focus away from an
external sovereign globalization-as-object to be grasped and wielded. Rather we must imagine a nonsovereign

production of the global that is as increasingly immanent in, and emergent through our day-to-day thoughts and

we don't
need to think of globalization as an abstraction because it is embedded in
everyday practices. We should therefore understand places, even those not deemed to be world cities,
actions as it is in the mass movement of capital, information and populations. According to Flusty,

and the activities of everyday life as sites in which globality is constituted and from which it is extended, and
marginalized groups of people around the world as practising revolutionary forms of insubordination even as they
engage in exploitative labour practices in order to survive. Furthermore, such people are really only excluded in
part from the global order because, as (potential) members of the multitude, they embody a double character of
poverty and possibility (Hardt and Negri 2004:129, 153). An immanent approach to the climate changeglobal
capitalism coupling is therefore crucial. I care about this thing we refer to as climate change but I refuse to

Climate change is entangled with the


contemporary dynamics of both capitalism and development and cannot
be considered in isolation from such dynamics. This is one of the reasons why
conventional scientific structures are struggling to handle the deep
scientific uncertainties which surround the probable effects of climate
change (Mustafa 2009:469; see also Foss 2009). Deleuze and Guattari's (1987) concept of the body without
organs (BwO) is a more productive way to think about climate change as an
immanent process. Such an approach enables us to retain our critique of global capitalism and
the harmful and oppressive molar forces it sets in motion, but it also facilitates an important focus
on everyday biopolitical production and on the molecular and affective
activities which break apart global orders. For Deleuze and Guattari (1987) the BwO is
a body populated by multiplicities in a continual process of becoming.
Climate change can only be understood in these terms. It is of course a vast
concept involving glaciers, polar bears, light bulbs, hurricanes, general
circulation models, building materials, trade, gender, presidents, forests,
celebrities, cities, conversations, TV programmes and movies, policy
documents, meetings, cow flatulence, insurance, transport, activism,
migration, food miles, beetles, malaria and many other heterogeneous
elements which are always enrolling new actants. We do need to both
expand and deconstruct the notion of climate change and although it could never be
understand it as an environmental issue.

modelled, the BwO is one possible way to welcome and better incorporate into our discourse and action the
molecular multiplicities which are part of the shifting practices, debates and policies that surround climate change.

The global climate model is a simplification of complex realities in order to


make scientific prediction, while the BwO embraces the complexities,
unpredictabilities and uncertainties, the flows and the disruptions. It
might prevent us from developing problematic molar, modernist and
localized desires to return to a world which we imagine to be less
connected and provide a more effective response to those trying to attend
to climate change while leaving neoliberalism intact . Such an approach might also
relieve scientists of the problematic need to try and talk about climate change with one voice out of fear that they
will be outed as frauds or conspiracy theorists by so-called climate change skeptics.5 Drawing on the ideas of

I would much rather be schizophrenic than paranoid about


climate change. An example of paranoia in climate change policy is the emergence of what Hume
(2009:103) has called technocratic models-for-policy and cites the example of how in 2005 Tony Blair
exhorted scientists to come up with the level of warming that must be
avoided and as a result we ended up with the 2 threshold around which a number
of NGOs, including Greenpeace and Christian Aid, began to campaign. Such a move reduces our
capacity to act flexibly, collectively and ethically and to understand
science as an ongoing fallible set of knowledge-making practices. It also
leaves us ill prepared for the moments when schizophrenic forms of desire
begin to subvert the coded and univocal 2 norm, which they surely will,
because as Deleuze and Guattari's work has shown, capitalism in its inevitable
propensity to deterritorialize and decode actually promotes
Deleuze and Guattari,

schizophrenia (Holland 1999:94). A schizophrenic approach, in contrast, would be far


less doggedly attached to a fixed technocratic approach, and would be
more open to local demands, desires and mobilities.

Global Warming Framework


1ACs universalizing spatial grammar of global warming
makes policy responses and public engagement
counterproductivemethodological investigation solves better
Brace and Geoghegan 11 Catherine Brace, University of Exeter, UK, and
Hilary Geoghegan, University of Exeter, UK, Human geographies of climate change:
Landscape, temporality, and lay knowledges, Progress in Human Geography, vol.
35, no. 3, pp. 284-302, June 2011, doi: 10.1177/0309132510376259
Climate has long been the domain of the natural scientist , often defined in
purely physical terms. It is only relatively recently that natural scientists, such as Hulme, have started

to suggest that climate is a quantity wholly disembodied from its multiple and contradictory meanings (Hulme,
2008: 6) and called for the intervention of social scientists to understand more about human impacts and responses
(Hulme, 2009; see Moser, 2010; Slocum, 2009). However, even a cursory glance at the pages of the journal Climatic
Change, to take but one example, will show that this work has been ongoing for some time through research which
attempts to describe, evaluate, quantify and model perceptions of climate change, understandings of risk and
construction of policy (to name but three topics; see, for example, Berk and Schulman, 1995; Jaeger et al., 1993;
Rayner and Malone, 1998; Smit et al., 1996). Although reconstructing the genealogy of climate change in the social
sciences and humanities is not the work of this paper, it is notable that back in 1993 Jaeger et al. were able to
identify work on the human dimensions of global climatic change, albeit at an early stage. More recently, Batterbury
(2008) sketches out a history of work on global warming in anthropology dating from the mid-1990s. Nevertheless,
despite the now substantial literature that is informed by the methods, theories and epistemologies of social

there is still a marked dearth of engagement with critical and cultural


theories which bring a different sort of interpretative leverage 1 to
questions about the human dimensions of climate change by focusing
(broadly speaking) on the way space, power, identity and knowledge constitute
social relations (exceptions, not all of which are focused on climate change but human-environment
science,

relations and knowledges more generally, are: Clark, 2000, 2005, 2007; Ingold, 2006; Ingold and Kurttila, 2000;
Slocum, 2004, 2009). Recent sessions at the Royal Geographical Society (with IBG) Annual Conference in 2009 on
such topics as Cultural Spaces of Climate and Geographies of the Seasons confirm the salience of a more critical
and contextual approach. Further, a recent special issue of the Journal of Historical Geography (Bravo, 2009; Daniels
and Endfield, 2009; Hamblyn, 2009; Liverman, 2009) marks a trend in histories of science towards histories of

there is now a demand (largely


from scientists and policy-makers frustrated with the lack of public
engagement and the concomitant failure of the deficit model 2 ) to
understand what climate change means to so-called ordinary people
climate science and change. In addition to scholarly manoeuvres,

(Bostrom et al., 1994; Hanski, 2008; Lorenzoni et al., 2007; Manzo, 2010). The relative neglect of this theme lies, as

the IPCC and other national and international


scientific bodies have tended to see atmospheric emissions as a universal
and global-scale problem affecting the climate system of the whole planet.
The result has been that the scientific study of the problem has been
decoupled from the social and political contexts of its material production
and cognitive understanding (Agrawal and Narain, 1991). Nevertheless, it is now recognized that
environmental knowledges, including those surrounding climate change,
need to be understood on a local scale. Researchers are slowly becoming
more interested in publics for whom popular representations of melting
polar ice caps and homeless polar bears have little currency and are far
removed from actual (possibly minimal) experiences of climate or climate change
(ONeill and Nicholson-Cole, 2009; Slocum, 2004; Wilbanks and Kates, 1999). Such climate
knowledges also need to be more carefully calibrated with phenomena
such as weather and the seasons which form part of the same lexicon but
are far from synonymous. Thus, using climate and the ways it may change
in preference to climate change enables a relational approach to emerge
Demeritt (2001a) notes, in the way that

which: (1) does not insist on research participants being able to


disentangle anthropogenic causes from natural causes of climate change ;
(2) acknowledges the way an understanding of climate change is conjoined
with other kinds of knowledge about the local environment; and (3) allows
different ways of knowing to play a legitimate part in framing a culture of
climate change. Thus lay knowledges of both climate and climate change
are considered important in this formulation. Useful though recent work on the emerging
cultures of climate change has been in identifying potentially fruitful directions for research which is more
grounded, it is ironically the ground that is missing from recent accounts. Landscape a central tenet of cultural

geography
is the natural home of research which takes a more interpretative
approach to climate change while at the same time offering a critical
reading of the natural sciences . . . informed by a spatially contingent view
of knowledge (Hulme, 2008: 5; see also Demeritt, 2009; Harrison et al., 2004). In this paper, we show that
geography for over 80 years is, curiously, elided. This is despite the claims of geographers that

there is now an opportunity to explore how individuals and communities understand climate and the ways it might
change in the context of local landscapes and environmental challenges, researched as a lived experience with a
unique set of geographies, lay knowledges, and participative practices. In so doing, we build on the work of Slocum
(2004) and Hinchliffe (1996) among others who have examined local environmental knowledge in the context of
everyday life. In this paper we foreground landscape as an organizing concept. We explore this in more detail in the
next section, before moving on to look at the importance of future-orientated temporalities and lay knowledges to
our focus on familiar landscape.

Critical investigation is key to crafting more responsible


responses to climate change
Head and Gibson 12 Lesley Head, University of Wollongong, Australia, and
Chris Gibson, University of Wollongong, Australia, Becoming differently modern:
Geographic contributions to a generative climate politics, Progress in Human
Geography, December 2012 vol. 36 no. 6 699-714

In this paper we identify and bring together several threads in recent human geography that could help shape
debate on climate change in the social sciences and more broadly. Human and environmental geographers have a

climate vulnerability
cannot be separated from underlying social and political dimensions (e.g.
Bohle et al., 1994). There are many geographic contributions to grounded studies
of place, local diversity and the difference this makes to issues like
adaptive capacity (Adger et al., 2005, 2009; Barnett and Campbell, 2010). Increasing
recognition of the social and cultural dimensions of climate change has led
to greater (and perhaps belated) interest in the sociocultural research tradition in
geography and cognate disciplines (e.g. Hulme, 2008; OBrien, 2011). Our specific argument is
that human geographys combination of both deconstructive and empirical
compulsions, found in co-existing emphases on critical theory and
ethnographic type research methods that focus on material, embodied
practices, provides a unique possibility to be both unsettling and
generative/creative. We believe this is exemplified best in the ideas of Gibson-Graham (2006, 2008),
long history of contributing to climate change discussions, and emphasising that

whose work we use to think through how to reframe the politics of climate change in response to, and beyond,

Gibson-Graham (2008) identify three practices that assist us here and


on which we attempt to build: ontological reframing to produce the
ground of possibility; rereading to uncover or excavate the possible;
creativity to generate actual possibilities where none formerly existed.
Ontological reframing is undertaken by engaging critically with stubbornly
persistent assumptions, norms and the taken-for-granted then looking
for ways to put together differently. We are keen here to consider how climate change can be
modernity.

reframed in this way by drawing attention to the persistence of the human/nature binary in climate change debates
(section II). We subsequently explore how relational frameworks can contribute to the reframing, putting knowledge
back together differently.

Rereading and creative generation then proceed through

fine-grained studies of local voices and practices, including identifying


vernacular capacities that could prove vital to climate change responses.
It is these voices and practices that a modernist vision of problem and
solution, scaled predominantly around the nation state, runs the risk of
ignoring. We bring together geographic work on governance, scale and
power that illuminates much more diverse pathways of agency (section III).
Importantly, geographic work on relational scale and cross-scalar agency offers pathways to re-empower the local
without reifying it as a pregiven subset of the global. As Gibson-Graham (2008: 3) remind us, to

change our
understanding is to change the world, in small and sometimes major
ways. A starting point is looking for productive or progressive spaces in
unlikely places (Lewis, 2009), to crack open new ways to converse. Throughout the
paper we draw on diverse examples where this can and might be done, using a refreshed conception of scale and
power that avoids locking down territories as containers of action. In the process of becoming, and becoming
understood as, a global problem, climate change has become recognised as a hybrid assemblage constituted as
more-than-climate, comprising discourses, bureaucracies and texts as well as atmospheric gases (Demeritt, 2001;
Hulme, 2008). The emerging critical analysis of climate-change-as-assemblage has much in common then with the
critique of related concepts like neoliberalism (Castree 2008a, 2008b; Dean, 1999; Peck, 2004), modernity and the
economy (Mitchell, 2002), colonialism (Anderson, 2007; Thomas, 1994) and capitalism (Gibson-Graham, 1996,

concepts have dominant, taken-for-granted meanings, but can be


analytically revealed as constituted through practices and discourses thus
inviting critiques that destabilise them and offer alternatives. There is a key
2006). Such

difference, however. The above scholarly critique has usually pulled the various threads of modern industrial
capitalism apart in order to imagine how subsistence might be constituted differently, and with more attention to

ontological status of categories such as


neoliberalism, the economy, capitalism and colonialism has been
challenged, in order to challenge their universalist power in other words to
contest and resist the concepts. Our approach to climate change is similar in epistemology but
different in intent. We want to contribute further to such deconstruction not to contest
the concept of climate change, but to suggest reconfigured responses. Indeed, we are
social and ecological justice. The

concerned as are others that academic deconstruction of the climate change assemblage may run the risk of
unwittingly buttressing reactionary sceptics and a range of vested interests. The step that has not yet been taken in
relation to climate change is to go from the ontological reframing to the generative possibilities. We seek here to
outline some possible directions.

AT: Warmings bad for the 3rd world


Paternalist frames of warming which emphasize humanitarian
consequences for the third world are biopolitically pacifying
and geopolitically stratifying
Cupples 12 Julie Cupples, Wild Globalization: The Biopolitics of Climate Change
and Global Capitalism on Nicaragua's Mosquito Coast, Antipode, Volume 44, Issue
1, pages 1030, January 2012, DOI: 10.1111/j.1467-8330.2010.00834.x

The accounts of the Atlantic Coast that we find in NGO and journalistic publications are on one level understandable
and defensible as they aim in part to call the first world, where most climate change emissions are produced, to

framings of indigenous peoples and climate change


generated by NGOs do however also function as a kind of biopower, identified by
Foucault (1978) as a force which through particular technologies and forms of
expertise defines, controls and regulates the life of populations in both
oppressive and life-enhancing ways.2 Like other forms of biopower enacted in modern societies,
such framings contain pastoral dimensions, but they also perpetuate
neocolonial fantasies of indigenous peoples as incapable of adapting to
modernity and as doomed to disappear. For Hardt and Negri (2000:36), humanitarian
NGOs such as Oxfam, which practise a kind of moral intervention are immersed
in the biopolitical context of the constitution of Empire and as a result
can be understood as some of the most powerful pacific weapons of the
new world order. As well, such accounts construe the human and nonhuman
inhabitants of the Coast and their complex geographies, economies and
ecologies as victims of the global, overlooking the creative tactics that are
developed in the context of such challenges. Consequently, the global is
mistaken for a force that is extraneously generated rather than one that
also resides in and extends from these localities (Flusty 2004). There is no discernible or
account and action. These

explicit attempt in such accounts to understand how such forms of biopower are contaminated by everyday forms
of biopolitical production in the Miskito Keys themselves, or to consider how the subjects targeted by biopower
engage in creative forms of biopolitics to resist this tendency (Corva 2009:172; see also Hardt and Negri 2004).

Hardt and Negri (2000:30) conceptualize biopolitics as a life politics, the living
development of productive labour in society which includes not just the
communicative and the intellectual, but also the corporeal and the
affective. This paper considers the conceptual gains that might accrue from tackling growing concerns about
climate change in the so-called third world through a focus on everyday immanent biopolitical production, as it is
developed in the work of Deleuze and Guattari (1987) and Hardt and Negri (2000, 2004).3 While a number of recent
publications have emphasized the need not to understand globalization in disempowering ways as something out
there to which locals can merely react but rather as a messy, contingent and unpredictable process in which global
power is frequently disrupted (Allen 2003; Flusty 2004; Hart 2002; Ong 1987; Tsing 2005), Hardt and Negri's
Deleuzoguattarian inspired approach is particularly useful for my case study because of the emphasis it places on
the immanent and biopolitical production of new figures of subjectivity, in both their exploitation and their
revolutionary potential (Hardt and Negri 2000:29). In addition, Deleuze and Guattari's emphasis on becoming
enables us to think beyond dominant approaches to climate change. Not only does it put capitalism more firmly in
the picture, but it also disrupts the concept of adaptation as something which people now need to do in response to

Becoming is perhaps more useful than the concept of adaptation, commonly


because it captures how the becoming of both
humans and nonhumans, in situations of economic exploitation and
degraded environments, is always in the process of adapting,
transforming and modifying itself in relation to its environment (Guattari
2008:90), forcing us to ask not what something is, but what it is turning into,
or might be capable of turning into (Bruun Jensen and Rdje 2009:1).
climate change.

used in debates about climate change,

Anda mungkin juga menyukai